Планиметрический дневник за 2010-2012

Николай Москвитин
 Планиметрический дневник.

Введение.
Данная книга называется «Планиметрический дневник». Это сборник задач, распределённых по датам в порядке своего создания авторам данной книги. Сначала публикуются все задачи автора за 2010-2012 годы, а затем даются все их решения. Книга разделена на три части, соответствующие трём годам написания задач. После всех частей даётся предметный указатель.

В напутствие вот такой «Гимн геометра» автора:

В мире нас много фигур окружает,
Столько гармонии в них, красоты,
Схемой в тетради их изображая,
Всмотримся в чудные эти черты:

В каждой фигуре скрываются связи,
Каждое свойство как чудный цветок,
В мире природа, а также и разум
Связями нам преподносят урок.
 
Людям поможем в практических нуждах
И в теоремах найдём красоту!
Скажем же ныне, геометры дружно:
Наша наука идёт в высоту!
22 февраля 2013

Желаю успеха в освоении науки планиметрия!
2010 год
Январь 2010
№1. Условие: треугольник ABC со сторонами AB=6, AC=9 и BC=5 ; окружность, касающаяся стороны AB в её вершине B  и стороны AC  - в точке D.
Найти: радиус этой окружности и расстояние от её центра до стороны BC.
9 января 2010.Николина Гора (числа были придуманы там же)
№2. Условие: треугольник ABC со сторонами AB=3, BC=5 и AC=7. Вокруг треугольника описана окружность и к ней в точке A проведена касательная.
Найти: расстояния от вершин B и C до этой касательной.
Январь 2010. Николина Гора

Задачи с неизвестной точной датой (но предположительно с февраля по май).
№3. Условие: прямоугольный треугольник ABC (угол B-прямой); вокруг треугольника описана окружность; квадрат BDEF с двумя соседними сторонами на лучах BC и BA и вершиной E на окр ужности. Меньший катет данного треугольника равен a, больший- b.
Найти: сторону квадрата BDEF.
№4. Условие: квадрат ABCD; полуокружность, построенная на стороне AD
как на диаметре. Из вершин B и С проведены касательные отрезки с основаниями E и F соответственно. BE пересекает CF в точке O.
Найти: BO:OE и CO:OF.
№5. Условие: прямоугольный треугольник ABC со сторонами a и b (AB=a; BC=b); окружность, касающаяся гипотенузы AC и стороны AB так, что её центр лежит на медиане BD.
Найти: радиус данной окружности.
№6. Условие: окружность с центром O; из точки С вне окружности под углом 120 градусов проведены касательные к окружности длиной a; через точку С проведена прямая, перпендикулярная одной из касательных.
Найти: длину отрезка CD (D-точка пересечения данной прямой с окружностью, более удалённая от С).




Июнь 2010 (июль 2010-?)

№7.Задача на построение.
 Условие: треугольник ABC. Известно, что высота BD образует со стороной BC угол в 45 градусов. Считается, что прямая BD, содержащая высоту, уже построена. Как всего одним движением циркуля построить ортоцентр треугольника ABC?

№8. Условие: ромб ABCD с острым углом ABC; около треугольника ABC описана окружность;
проведены диагонали AC и BD. Окружность пересекает диагональ BD в точке K.
Доказать: прямая CK перпендикулярна стороне AD.

№9. Условие: окружность.
Найти: геометрическое место оснований касательных отрезков из точки  большой окружности к меньшим концентрическим окружностям.

№10. Условие: прямоугольный треугольник ABC. ( угол B- прямой) .Биссектриса AD. Высота BE. Биссектрисы углов ABE и EBC- BF и BG соответственно. Они пересекают биссектрису AD в точках J и I соответственно.
Доказать: ;;=IJ=IG=IE.
Июнь или июль 2010, Москва

Август 2010
№11. Условие: две параллельные прямые a и b; полуокружности, касающиеся друг друга внешним образом  внутри параллельных прямых с диаметрами на данных прямых (a и b) – AB и CD соответственно (точка касания — O).
Доказать: A, O, D лежат на одной прямой так же, как и B, O, C, причём прямые AD и BC перпендикулярны.
№12.Условие: равнобедренная трапеция ABCD; диагонали AC и BD пересекаются в точке O; в трапецию можно вписать окружность.
Доказать: угол BOC>60 градусов.
Осень 2010
№13.Условие: на диаметре AB окружности выбрана произвольно точка D. Перпендикуляр к AB, проведенный через точку D, пересекает окружность в точке C. На AD и DB как на диаметрах построены окружности. Общая касательная к этим окружностям, пересекающая CD, касается их в точках M и N соответственно.
Доказать: отрезок AC проходит через точку M, а BC - через N.
 Сентябрь 2010, Москва.
№14. Условие: квадрат ABCD. На AD как на стороне внутрь квадрата построен правильный треугольник ADE. Диагональ AC  пересекает сторону ED этого треугольника в точке F.
Доказать: CE=CF.
№15. Условие: в квадрате ABCD отмечены середины двух соседних сторон BC и CD- Е и F соответственно- и проведены прямые AE и BF, пересекающиеся в точке G. Около квадрата описана окружность. Точка пересечения прямой AE с нею- точка H.

Доказать: GE=EH.
№16. Условие: равнобедренная трапеция. В вершинах тупых углов восстановлены перпендикулярные прямые к боковым сторонам трапеции.
Доказать: угол, образуемый этими прямыми (с лучами, направленными к вершинам трапеции), в два раза больше острого угла трапеции.

№17. Условие: две окружности, внутренно касающиеся друг друга (точка касания- A); из точки большой окружности B, диаметрально противоположной точке их касания, проведена касательная к меньшей окружности (точка касания- С); прямая BC пересекает большую окружность в точке D, прямая AC - в точке E.

Доказать: дуги DE и BE равны.
Ноябрь 2010
№18. Условие: равнобедренный прямоугольный треугольник ABC (угол С-прямой); вписанная и описанная окружности; вписанная с центром O касается стороны BC в точке E; биссектриса угла ; пересекает описанную окружность в точке F.
Доказать: OE=EF.
Декабрь 2010
№19.Условие: равнобедренный треугольник ABC с основанием AC.
Доказать: конец отрезка, выходящего из вершины B, параллельного основанию и равного боковой стороне треугольника, является центром вневписанной окружности треугольника.
№20. Задача на построение.
Условие: отрезок AB; на этом отрезке взята произвольная точка С; на полученных отрезках как на сторонах построены квадраты; известно, что все их вершины лежат по одну сторону от отрезка.

Постройте с помощью одной линейки квадрат, диагональю которого является отрезок AB.
№21.Условие: вокруг квадрата ABCD описана окружность.
Доказать: луч света, посланный из вершины B в произвольную точку E дуги CD, отразившись от прямой CE, попадёт в вершину D.

2011 год.
Январь 2011
№1. Условие: в равнобедренной трапеции ABCD диагонали AC и BD пересекаются в точке O и  равны основанию AD.
Доказать: центр вписанной в треугольник AOD окружности совпадает с центром описанной окружности трапеции.
3 января Николина гора
№2. Условие: меньшее основание BC трапеции ABCD относится к большему основанию AD как 1:2. В трапеции проведена высота DE к боковой стороне AB.
Доказать: CE=CD
5 января Николина гора
№3.Условие: равнобедренная трапеция ABCD. Диагонали AC и BD пересекаются в точке O.
Около треугольника AOB описана окружность, пересекающая основание  AD в точке E.
Доказать: расстояние от точки O до прямой  BE равно разности высот треугольников ABD и AOD, проведённых к AD.
7 января Николина гора
№4.Условие: равносторонний пятиугольник с двумя прямыми углами- ABC и AED. Диагонали его BD и CE пересекаются в точке F.
Доказать: отрезок FA равен стороне пятиугольника.
8 января Николина гора
№5.
Условие: вокруг равнобедренного треугольника ABC с основанием AC описана окружность. В треугольнике проведены все высоты. Точка их пересечения- F. Высота CE продолжена до пересечения с данной окружностью в точке G. Около треугольника GBF описана окружность.

Доказать: Высота AD является касательной для второй окружности (точка касания - ортоцентр треугольника).
15 января Москва, вечер.

Февраль 2011
№6.Условие: равнобедренный треугольник ABC (AB=BC). В треугольнике проведены все высоты: AD, BE и CF. Также в прямоугольном треугольнике BEC  проведена высота EG.
Доказать: прямые FD и EG пересекаются на описанной окружности треугольника BEC.
3 февраля Москва
№7.Условие: около треугольника ABC описана окружность и середины дуг E, F,G, на которые её разбивают её вершины, попарно соединены. Биссектрисы треугольника ABC продолжены до этих точек. Точки пересечения сторон второго треугольника с ними K,L,M также попарно  соединены и через отрезки KL, LM и KM проведены прямые.
Доказать, что они отсекают от сторон треугольника попарно три ромба.
4 февраля Москва
№8.
Условие: квадрат ABCD. Дуга окружности с радиусом, равным стороне квадрата, с центром в точке С, заключённая между сторонами квадрата. К ней проведена произвольная касательная, пересекающая стороны AD  и AB в точках E и F соответственно. Диагональ BD. Отрезок CF, пересекающий её в точке G. Прямая EG пересекает сторону BC в точке H.
Доказать: отрезки CF и EH перпендикулярны и равны.
9 февраля Москва
№9.Условие: квадрат ABCD. Полуокружность, построенная на стороне AD, как на диаметре.
К ней из точки B проведена касательная BE и через точку касания проведены прямые AE и DE, пересекающие стороны CD и BC в точках K и L соответственно.
Доказать: отрезок KL параллелен диагонали BD.
15 февраля
№10.Условие: две равных окружности касаются друг друга внешним образом. Через точку касания проведены произвольные перпендикулярные прямые.
Доказать: отрезок, соединяющий точки пересечения этих прямых с окружностями, расположенные по одну сторону от линии центров, параллелен их общей внешней касательной и равен диаметру окружностей.
20 февраля Москва
№11.
Условие: на сторонах квадрата ABCD AB и BC отмечены соответственно середины M и N. Около квадрата описана окружность. Прямые DM и DN пересекают эту окружность соответственно в точках K и L.
Доказать: отрезок KL делится сторонами квадрата на три равных отрезка.
Февраль 2011
Апрель 2011
№12.Условие: окружность с центром O. Хорда AB продолжена  до точки C так, что касательный отрезок CD равен AD.  Радиус OD пересекает прямую AB  в точке E.
Доказать: BE=BC.
18 апреля
№13.Условие: треугольник ABC; описанная окружность с центром O; проведены радиусы её к вершинам. Обозначены центры описанных окружностей полученных треугольников: O1,O2,O3.
Доказать: O является центром вписанной окружности O1O2O3.
20 апреля
№14.Условие: трапеция ABCD- равнобедренная.
Доказать: Z=R(x-y)/b=(x^2-y^2)/2h, где z-расстояние от центра описанной окружности трапеции до точки пересечения диагоналей, x и у- отрезки, на которые она делит их, b-боковая сторона, R-радиус описанной окружности, h-высота.
24 апреля, Москва
Май 2011
№15.Задача на построение.
Условие: равносторонний треугольник, на двух его сторонах отмечены их середины.
Построить центр вневписанной окружности с помощью одной линейки.
19 мая
Май-июнь 2011
№16.Условие: в равнобедренной трапеции ABCD c основаниями AD и BC (меньшее основание) радиус описанной окружности OB параллелен боковой стороне CD.
Доказать: расстояние от центра описанной окружности трапеции до любой диагонали равно  половине меньшего основания.
Июнь 2011
№17. Условие: прямоугольный треугольник ABC (B-прямой). Высота AD, биссектриса CE. Пересекаются в точке F.
Доказать: если отрезок AE перенести параллельным переносом так, что A совместится с F, E  окажется на гипотенузе.
1 июня 2011

№18.
Условие: треугольник ABC. На его  стороне AC отмечено основание биссектрисы угла треугольника B-D. Через это основание и вершину B проведена окружность, касающаяся стороны AC
Доказать: отрезок, соединяющий точки  пересечения данной окружности с двумя другими сторонами треугольника, параллелен его стороне AC.
4-5 июня

№19. Условие: треугольник ABC; высота AD; биссектрисы углов, на которые разбивает она угол A, пересекают сторону BC в точках E,F (первая лежит на BD, вторая  на CD), около треугольника EAF описана окружность. Окружность пересекает стороны AB и AC в точках G и H соответственно.
Доказать: прямые AD, EH и FG  пересекаются в одной точке.
№20.
Четырёхугольник ABCD с диагоналями AC, BD, прямыми углами ABC и ADC. Обладает свойством: биссектриса угла  CAB – AE-параллельна стороне CD. Она пересекает диагональ BD в точке F. Из точки F восстановлен перпендикуляр к EF, пересекающий CD в точке G.
Доказать: G-центр описанной окружности треугольника AEС.
17 июня
№21.
Докажите, что в равноугольно-полуправильном шестиугольнике  диагонали, соединяющие вершины через две, являются биссектрисами углов, из которых они выходят. Покажите, что их периметр равен периметру данного шестиугольника.
 22-24(?) июня
Июнь-июль 2011
№22. Доказать, что радиус описанной окружности равнобедренной трапеции, имеющий конец в одной из вершин малого её основания, параллелен её боковой стороне тогда и только тогда, когда угол между одной парой радиусов (проведённых к вершинам большого основания) в три раза больше угла между другой парой радиусов (проведённых к вершинам меньшего основания).
Июнь 2011
№23.Доказать, что в равнобедренной трапеции с перпендикулярными диагоналями, равными большему основанию, радиус описанной окружности, проведённый к одной из вершин противоположного основания, параллелен боковой стороне трапеции.
30июня-1июля 2011.
Июль 2012
№24.Условие: неравнобедренный треугольник ABC. Биссектриса BD. Перпендикуляры к боковым сторонам AB и BC из точки D-соответственно DE и DF. Прямая EF пересекает прямую AC  в точке  в точке G так, что DA=AG. Стороны AB и BC равны a и b соответственно.
Найти: sin ABC.
2-3 июля
№25. Условие: окружности с центрами O и O1-соотвественно  w1 и w2- касаются друг друга. Из точки A вне окружностей проведены к  ним равные касательные AB и AC ( B принадлежит w1, C принадлежит w2). BС вторично пересекает окружность w2 в точке K. Доказать:O1K параллелен OB.
4-5 июля
№26. Докажите, что расстояние между центрами для всех пар окружностей, вписанных в ромб так, что одна касается другой и двух его пересекающихся сторон (при этом их точки касания лежат на разных сторонах), постоянно.
7-8 июля
№27. Условие: в прямоугольнике проведены диагонали, и в каждый образовавшийся треугольник вписана окружность.
Доказать, что сумма их радиусов меньше половины периметра прямоугольника.
10 июля
№28.
Условие: в прямоугольном треугольнике ABC с прямым углом B и углом BAC<30 внутри его взята «необычная» точка E, такая, что EC=BC=AE, проведён перпендикуляр BD к гипотенузе AC.
Докажите, что ED=DC
10 июля
№29. Условие: к окружности с центром O проведены касательные из точки A –AB и AC. Проведена хорда BC. В образовавшемся треугольнике ABC проведены высоты CD и BE, пересекающиеся в точке F.
Доказать: отрезок OF делится BC  пополам.
11 июля
№30. Условие: две окружности с центрами O и O1 касаются внешним образом. Проведена общая касательная MN. Из O  восстановлен перпендикуляр к линии центров, и точка K его пересечения с прямой MN соединена с O1. Оказалось, что O1K-касательная к окружности с центром O. .
Найти соотношение радиусов окружностей (меньшей к большей).
Ответ: 1:2
12 июля 2011
№31. Условие: окружности с центрами O и O1 (первая большая) касаются внешним  образом в точке D. Из точки A большей окружности проведена касательная к меньшей окружности (точка касания C) так, что расстояние от центра большей окружности (O) до прямой AС равно радиусу меньшей. AC вторично пересекает большую окружность в точке B. Середина хорды AB- точка E-соединена с O1, причём EO1=scrt2.
Найти: радиус описанной окружности треугольника ODC.
15 июля
№32. Условие: на листе бумаги нарисовали две концентрические окружности (их общий центр- O). Из O провели радиус OA большей окружности и из A провели касательную   к малой окружности AB, затем через  B провели радиус OC и эту операцию продолжили  до тех пор, пока цепочка не замкнулась, то есть основание очередной касательной совпало с  точкой пересечения радиуса OA и малой окружности. Назовём такую фигуру (т.е. получившийся многоугольник) «солнышком». Пусть у солнышка 2 n зубьев.
Докажите, что площадь такого солнышка равна площади вписанного в большую окружность n-угольника, стороны которого соединяют его зубья через один.
18 июля
№33.
Существует ли треугольник, который можно путём проведения двух прямых из   вершины разрезать на три равнобедренных?
18 июля
№34.Условие: каждую из сторон произвольного треугольника разделили на три равные части. Каждую вершину и более удалённую точку деления отметили разными цветами (соответственно получилось по три точки каждого из цветов на каждых двух соседних сторонах). На отрезках, концами которых являются точки одного цвета, как на диаметрах, построили полуокружности.
Докажите, что три точки их пересечения образуют вершины правильного треугольника.
18 июля конец дня (17-ый час)
№35. Условие: три окружности с центрами O1,O2,O3 касаются друг друга. Стороны O1O2, O3O2, O3O1 продолжены до пересечения с окружностями c центрами O1 и O2: прямые O1O2 и O3O2 пересекают окружность с центром O2 в точках A1 и A2 соответственно, прямые O1O2 и O 3O1 пересекают окружность с центром O1 в точках C1 и C2 соответственно. Прямые C1C2 и  A1A2 пересекаются в точке B. Известно, что угол O1O3O2 равен 60 градусам.
Найти: угол  C1BA1.
19 июля

№36.  Условие: два обычных школьных угольника с углом 45 градусов и одинаковыми сторонами совместили так, что вершина одного оказалась на середине гипотенузе другого, и лучи первого прошли через две вершины второго.
Найдите площади заштрихованных на рисунке частей (считается, что угольники абсолютно плоские).

 
19 июля 2011 (идея старее)

№37.
Условие: два равных школьных угольника с углом 45 градусов и с равной шириной рамки наложили друг на друга так, что вершина каждого оказалась на середине диагонали другого. В результате образовались в том числе квадрат и четыре равнобедренных прямоугольных треугольника. Пусть их сумма равна квадрату( хотя это и отличается от стандарта; см. рисунок). Длина катета каждого из угольников равна a. Найти: ширину рамки.

 

Август 2011
№38. Условие: в треугольнике ABC известно, что BC<AB<AC  и что AC<R*scrt(2+scrt3), где R-радиус описанной окружности. В треугольнике проведены высоты BD и AE, отмечены их точка пересечения G и центр описанной окружности O.
Сравните углы AGB  и AOС.
№39. Условие: равнобедренная трапеция ABCD(AB=CD). Диагонали AC и BD пересекаются в точке O. Оказалось, что окружность, описанная около   треугольника AOD, касается AC и BD в точках A и D соответственно.
Докажите, что каждая из сторон AB, BC, CD меньше AD/3.
9 августа
№40. Условие: в окружности с центром O проведена хорда AB; на ней отмечена точка C. На отрезке BC как на диаметре построена окружность, пересекающая большую окружность в точке D, а диаметр из точки B большей окружности- в точке E. Оказалось, что BD=AD.
Доказать: AE параллелен BD.

9 августа
№41. Условие: определите все возможные виды четырёхугольников, у которых отношение двух сторон равно отношению двух других сторон.
23 августа

№42.
 Условие: четырёхугольник ABCD-вписанный, BD-диаметр, AD+CD=a, BD=d, угол ABC=a.
Найти: стороны четырёхугольника.


Октябрь 2011
№43.
Условие: квадраты ABCD и EFGH расположены так, что сторона EH лежит на AD, их середины совпадают, и один квадрат находится внутри другого.  Диагональ BD пересекает FG в точке J, а GH-в точке K.  FG продолжен до пересечения с AB в точке I. Отрезок KI пересекает EF в точке N.
Доказать: E,N,J,K лежат на одной окружности.
22 октября
№44. Условие: треугольник ABC. Из середин сторон AB и BC- D и F соответственно восстановлены перепендикуляры к ним, пересекающие сторону AC в точках E и G соответственно.  Центр описанной окружности O соединён с вершинами A и C. Точки пересечения прямых BE и BG с радиусами OA и OC соответственно- H и I.
Доказать: HI параллелен AC.
26 октября
№45. Условие:  треугольник ABC;  D-середина AB;  из D восстановлен серединный перпендикуляр к   AB, пересекающий сторону AC (для определённости) в точке F; средняя линия DE, E-середина AC. Центр описанной окружности O.  OB  пересекает  DE в точке G.
 Доказать: FG перпендикулярен OB.
26 октября 2011
Ноябрь 2011
№46. Условие: четырёхугольник ABCD-вписанный.  Прямая , параллельная CD, пересекает стороны AD и BC в точках E и
F соответственно. Прямые  AF и BD  пересекаются в точке H, прямые AC  и BE- в точке I.
Доказать: HI параллелен CD.
4 ноября

№47. Условие: равнобедренный треугольник ABC.  Из середин сторон AB и BC- D и E соответственно- восстановлены серединные  перпендикуляры, пересекающие сторону AC в точках F и G соответственно. Проведены радиусы  OA и OC.  Проведены прямые BF и BG, пересекающие их соответственно в точках H и I. J- середина  AC.
Доказать:  I,J,D лежат на одной прямой.
4 ноября
№48. Условие: прямоугольный треугольник ABC с прямым углом B; средняя линия DE (середины лежат на катетах). BF- высота, пересекает DE в точке G, на средней линии взята точка H так, что HE=DG.
Доказать:  AH=CH.
6 ноября
№49. Условие: треугольник ABC- равнобедренный (AB=BC). На стороне AB взята произвольная точка D. В треугольник BDC вписана окружность с центром K.
Доказать: A, D, K, С лежат на одной окружности.
 (комментарий: вроде 2 января 2012 последняя версия, но начало конца 2011)
№50. Условие: треугольник ABC; медиана BD; на ней взята точка E, такая, что угол DCE  равен углу  DBC; Проведены прямые CE и AE, пересекающие стороны AB и BC в точках F и G соответственно.
Доказать: FG параллелен стороне AC.

№51. Условие:  остроугольный равнобедренный треугольник ABC с основанием AC. Из B проведены два луча, образующие равные углы с боковыми сторонами треугольника и пересекающие продолжения AC в точках D и E, так, что угол DBE равен углу BAC. В треугольнике  BDC проведена высота DF, пересекающая высоту BG в точке O.
Доказать:  O- центр описанной окружности DBE.
Предположительно декабрь 2011
№52. Условие: равнобедренный прямоугольный треугольник ABC с вершиной B. На стороне AC взята точка D и на AD как на катете построен равнобедренный прямоугольный треугольник ADE.
Доказать: прямая AD делит EC пополам.
Декабрь 2011

2012 год.
Январь 2012.
№1.
Условие: в равнобедренной трапеции ABCD (;;=CD) проведена высота BG, и на боковых сторонах трапеции AB и CD отмечены их середины- E и F соответственно. Угол EBF-неострый.
Доказать: BG<(BC+AD)/2.
10 января и 16 января
№2.
Доказать, что существует вписанный четырёхугольник, который можно разрезать на три попарно неравных подобных равнобедренных треугольника.
Редакция 31 августа 2012; редакция сентября 2012.
№3.
Условие:  в окружности с центром O проведён диаметр AB, на ней взяты точки С и E, из них опущены перпендикуляры на диаметр AB- CD и EF соотвественно, причём CD=2EF. Отрезок BE пересекает CD в точке G. Через С проведена хорда CH, равная CD.
Доказать: A,G,H лежат на одной прямой.
Февраль 2012.
№4. Задача на построение.
Условие: дана окружность без центра, хорда AB, произвольная точка C на этой хорде, из C восстановлен перпендикуляр, пересекающий окружность в точке D, причём CD- больший из двух возможных. Построить циркулем и линейкой за два шага такую точку E на дуге AB, что EF, перпендикулярная AB, высекала бы на окружности хорду, равную CD.
№5. Условие: на сторонах BC и AD прямоугольника ABCD взяты точки A1,B1,C1,D1 (соответственно A1 и D1 на стороне AD, B1 и С1- на стороне BC), являющиеся вершинами квадрата, причём его центр совпадает с центром прямоугольника); прямая A1C1 пересекает прямую AB в точке ;, прямая B1D1- CВ в точке F. Отрезок С1E пересекает диагональ BD в точке G.
Доказать: точки E,D1,G лежат на одной прямой.
№6. Условие: дан прямой угол A, в него вписана окружность. В окружности проведены два перпендикулярных радиуса- OB и OC, касательная в точке B пересекает один из лучей в точке D, а касательная в точке С пересекает другой луч в точке E.
Доказать: D,O,E лежат на одной прямой.
23 февраля
Март 2012.
№7. Условие: четырёхугольник ABCD. Вокруг треугольников BCD и ABD описаны окружности, пересекающие AD и BC в точках E и F соответственно (соотвественно окр.1 и окр.2); окр.2 пересекает диагональ четырёхугольника AC в точке G; DF пересекает CE в точке H;  окр.1 пересекает диагональ AC в точке I; BE пересекает AF в точке K.
Доказать:  KI||GH.
17 марта и  23 марта
№8.
 Условие: вне окружности с центром O взята точка  A, такая, что касательные к окружности из неё AB и AC образуют угол в 45 градусов; AO пересекает окружность в точке G. Точка D на AB, такая, что OB=BD; около ADC описана окружность, пересекающая  первую окружность в точке E, а AO- в точке F; O1- центр этой окружности.
Доказать: 1) C,G,D лежат на одной прямой; 2) OF=EF; 3) OO1|| AB.
1) 22 января 2) 30 марта 3) 31 марта.
Апрель 2012
№9.
Условие: стороны прямоугольника ABCD  относятся как 2:3 (BC:CD=2:3); на CD взята точка E так, что BC=CE. F-середина AD; проведены отрезки BF, CF, BE и AG: BG-перпендикуляр из B  на CF, AE пересекает BF в точкеI, BE пересекает AG в точке K; IK пересекает BC в точке N .
Доказать: 1) AE- биссектриса угла GAD 2)N,G,E лежат на одной прямой.
6,7,8 апреля
№10.
Условие: на основании AD равнобедренной трапеции ABCD отмечена середина E; вокруг треугольника ABE описана окружность, пересекающая CE в точке F, BF продолжен до пересечения с прямой AD  в точке L, около EFL описана окружность с центром ;. Диаметр FF1 пересекает прямую AD   в точке K, AF пересекает  линию симметрии трапеции в точке J.  Высота трапеции B; пересекает прямую LO  в точке P, а линию симметрии  трапеции– в точке Q; AG- диаметр.
Доказать: 1) JK||BF; 2) P,G,F,Q  лежат на одной окружности
6-8, 15 апреля
№11.
Условие: окружность с центром O; из точки A вне окружности проведены касательные к окружности AB и AC; OA пересекает окружность в точке D; DE- касательная, E принадлежит AB; OE пересекает BC в точке F; BC пересекает AO в точке G. Точка B1, диаметрально противоположная B, соединенная с D, отрезок DB1 пересекает BC в точке H.
Доказать: CH=2FG.
20 апреля
№12.
Условие: в треугольнике ABC проведены высоты CD и AE, пересекающиеся в точке H; на стороне BC взята её середина G, проведены отрезки DG и EG. Биссектрисы углов DGE и ABC пересекаются в точке K.
Доказать: HK перпендикулярен BK.
22 апреля

№13. Задача на построение.
Условие: некоторая прямая пересекает стороны AB и BC треугольника в точках E и F; надо построить точку касания окружности со стороной AC, которая бы не только касалась стороны AC, но и пересекала бы AB и BC в таких точках  G и H, что GH параллелен EF.
30 апреля 2012
Май 2012
№14.
Условие: в треугольнике ABC отмечен центр вписанной окружности I, проведены отрезки CI,BI, AI, так, что отрезок CI численно равен произведению AI*BI, из I опущен перпендикуляр ID на BC.
Доказать: если CD=a, SABC>3a/2.
№15.
Условие: равнобедренный треугольник ABС, на стороне BC взята точка D так, что перпендикуляр к BC в этой точке пересекает AB в такой точке E, что DE=AE.
Найти: угол DAC.
6 мая
№16.
Условие: треугольник ABC- остроугольный; проведены высоты AA1, BB1, CC1, ортоцентр H. Около треугольника AHB описана окружность с центром O1.
Доказать: вершина С, центр окружности Эйлера E и O1 лежат на одной прямой.
8 мая.
Июнь 2012
№17.
Условие: в окружности с центром O проведены хорда AB, диаметр AC хорда BC; в сегмент, ограниченный AB и меньшей дугой окружности, вписана окружность с центром I;  прямая CI пересекает меньшую окружность в точке D, большую - в точке E. К меньшей окружности в точке D проведена касательная, пересекающая хорду AB в точке F; на меньшей дуге окружности BE взята середина G, прямые FI и CG пересекаются в точке H; BD пересекает FH в точке J.  Точка касания меньшей окружности и хорды AB- K. На CH выбрана точка L, такая, что, если провести перпендикуляр из I на прямую FL-IM, то угол ILM равен углу MBJ.
Доказать: FD, BH и KM пересекаются в одной точке.
1-4 июня
№18.
Условие: треугольник ABC, для которого BC=(AB+AC)/2. Высоты BB1 и CC1, ортоцентр H, точка Нагеля N.
Доказать: HN является биссектрисой угла B1HC (вариант задачи: N лежит на биссектрисе угла B1HC).
2 июня
№19.
Условие: две окружности касаются внутренним образом в точке A; центры их- O1 и O2 (первая-меньшая, вторая-большая), на касательной к окружностям в точке отмечена точка N, из N проведена касательные к большей и меньшей окружности- NC и NB соответственно; прямая BC пересекает прямую NO1 в точке L, NO1 пересекает большую окружность в точках K и P.
Доказать: PL=LK. На основе темы 2011, никуда не вошедшей.
6 июня
№20.
Условие: вокруг квадрата описана окружность с центром O, проведены диагонали AC и BD; ; на дуге CD взята произвольная точка G и в ней к окружности проведены касательная. Диагонали BD и AC пересекают её в точках E и H соответственно. Из E проведена вторая касательная к описанной окружности, точка касания- F.  BF пересекает диагональ AC в точке I.
Доказать: углы GID и IHD равны
7 июня на основе темы 2011
№21.
Условие: квадрат ABCD. Дуга окружности с радиусом, равным стороне квадрата, с центром в точке С, заключённая между сторонами квадрата. К ней проведена произвольная касательная, пересекающая стороны AD  и AB в точках E и F соответственно. Диагональ BD. Отрезок CF, пересекающий её в точке G. EG пересекает CM в точке K; CE пересекает BD в точке L.
Доказать: F,K,L лежат на одной прямой.
№22.
Условие: ABCD-вписанный и не является прямоугольником; его противолежащие стороны AB и DC продолжены до пересечения в точке E; BC и AD продолжены до пересечения в точке F; в треугольнике AED отмечен цент вписанной окружности I; из точки F проведён луч, делящий угол AFB в отношении 1:3.
Доказать: этот луч перпендикулярен биссектрисе угла AID.
10 июня, решая задачу Скутина
№23.
Условие: треугольник ABC. Из середин сторон AB и BC- D и F соответственно восстановлены перпендикуляры к ним, пересекающие сторону AC в точках E и G соответственно.  Центр описанной окружности O соединён с вершинами A и C. Точки пересечения прямых BE и BG с радиусами OA и OC соответственно- H и I; OB пересекает AC в точке J; IJ и HJ пересекают прямые OD и OF в точках K и L соответственно.
Доказать:  OB, KL и DF пересекаются в одной точке.  13 июня.
№24.
Условие: диагонали прямоугольника ABCD пересекаются в точке E. С центром в точке E построена окружность так, что они находится внутри треугольника. Из вершин C,D,A проведены касательные к окружности CF,DG, AH; СF пересекает DG в точке I, EI пересекает AD в точке J; прямые AH и CF пересекаются в точке L.
Доказать: отрезок LJ перпендикулярен AD.
19 июня, редакция сентября 2012
№25.
Условие: на сторонах треугольника с периметром p построены произвольные неостроугольные треугольники.
Доказать: периметр треугольника, вершинами которого являются вершины этих треугольников, отличные от вершин данного, меньше 3p/2.
24 июня
№26.
Условие: в треугольнике ABC проведена высота  AD,  биссектрисы треугольников BDA и CDA- AE и AF соответственно (E принадлежит BD, F- CD).
Доказать: биссектриса угла A проходит через центр описанной окружности EAF.
24 июня
№27.
Условие: на сторонах AB и BC треугольника ABC отмечены их середины- D и E соответственно, к ним проведены серединные перпендикуляры OD и OE (O-центр описанной окружности), они пересекают сторону AC в точках F и G соответственно; на BG взята точка H так, что BF=BH.
Доказать: F,H,G и O лежат на одной окружности.
26 июня, на основе темы 8 ноября 2011
Июль 2012
№28.
Условие: в треугольнике ABC проведены высоты AA1, BB1, CC1, отмечен ортоцентр H; проведена прямая l, параллельная  AC,  пересекающая отрезок от H до этой же стороны; относительно неё отражён ортоцентр и получена точка H’. Точка пересечения прямой l и высоты CC1- точка D, прямая AH’ пересекает прямую l в точке E, прямая DH’ пересекает прямую AC в точке F, отрезок FB пересекает прямую l в точке J, JH пересекает CE в точке K.
Доказать: JK перпендикулярен BE.
13 июля, Николина Гора

№29. Задача на построение.
Условие: на стороне AB треугольника ABC как на хорде построена окружность; найти такую точку на ней внутри треугольника, чтобы касательная, проведённая к окружности в этой точке, отсекала бы на сторонах BC и AC такие точки D и E, что A,B,D,E лежат на одной окружности
20 июля, Николина Гора

№30.
Условие: чевиана BD в треугольнике ABC, центры описанных окружностей треугольников ABD и CBD- O1 и O2 соответственно, высота BE.
Доказать: площадь четырёхугольника EO1BO2 равна половине площади треугольника ABC.
21 июля, Николина Гора
Август 2012
№31. Условие: угол с вершиной A; на отрезке луча AB как на диаметре построена полуокружность с центром O, пересекающая другой его луч в точке E; из некоторой точки C луча AE проведена касательная к окружности CD. AD пересекает BE в точке F,OD пересекает BE в точке G, СF пересекает AG в точке H, касательная к окружности в точке A пересекает прямую CD в точке I; AG продолжен до пересечения с окружностью в точке J; IH пересекает OJ в точке K;
Доказать: углы AKO и OID равны
12 августа на основе темы 2011
№32.
Условие: равносторонние треугольники ABC и CDE, у которых вершины A,C,E лежат на одной прямой, а все другие вершины лежат по одну сторону от отрезка AE; около треугольников х-описаны окружности, пересекающиеся в точке F. Центры описанных окружностей O1 и O2- соответственно в порядке, в котором треугольники даны в условии.
Доказать: 1)A,F,D лежат на одной прямой; 2) Пусть O1O2 пересекает AD в точке K; тогда AK=BF
13 августа, Москва, до 14:00
№33.
Условие: в треугольнике ABC проведены высоты AA1,BB1,CC1, на окружности, описанной около треугольника ABC, отмечена произвольная точка M (при условии, что она не принадлежит ни одной прямой, содержащей высоту; в последнем случае одна из окружностей вырождается в прямую).
Доказать: окружности, описанные около треугольников MAA1,MBB1 и MCC1, пересекаются в одной точке.
25 августа
№34.
Доказать, что если один из углов треугольника равен p/6, то отрезки, соединяющие центр его окружности Эйлера с концами стороны, противолежащей этому углу, перпендикулярны.         
 29 августа

Сентябрь 2012
№35.
Условие: вокруг остроугольного треугольника ABC описана окружность; проведены высоты AA1 и BB1, отрезок A1B1 пересекает окружность в точках D и E. BC=a, угол BAC=b, BB1=h0.
Доказать: 1)DE= 2sqrt((a-ho)*(acos^2b+hosin^2b))
1 сентября
№36.
Условие: вокруг треугольника ABC описана окружность с центром O. Через центр описанной окружности проведена прямая, отсекающая на сторонах BC и AC такие точки D и E, что A, B, D, E лежат на одной окружности. BD=a, CD=b.
Найти: радиус окружности.
2 сентября
№37.
Условие: равнобедренный треугольник с углом при общей вершине боковых сторон p/6. Центры описанной окружности и окружности Эйлера- O и E  соответственно.
Найти: угол OAE.
4 сентября
№38.
Условие:   в треугольнике ABC проведена  биссектриса BD, в точке D к стороне AC восстановлен перпендикуляр, на нём выбрана точка E, из которой эта сторона видна под прямым углом.
Доказать: основание биссектрисы треугольника AEC, проведённой из точки E, совпадает с основанием симедианы треугольника ABC.                6 сентября, вечер
№39..
Условие: окружность, хорды BA и BC; известно, что угол ABC= b, а BA+BC= a. Хорда BD, делящая угол ABC пополам.
Найти: длину этой хорды через a и b.
6-7 сентября
№40.
-Условие: в треугольнике ABC проведены высоты AA1, BB1, CC1, построена окружность Эйлера. Из ортоцентра H опущены перпендикуляры на стороны ортотреугольника A1C1, A1B1 и B1C1- K,M и L соответственно, и их основания соединены с вершинами треугольника.
Доказать: окружность, описанная около треугольника, образованного серединами отрезков AL, BK и CM,  концентрична окружности Эйлера и содержит её.
7-8 сентября
№41.
Условие: в четырёхугольнике известно, что две противолежащие стороны равны и суммы квадратов противолежащих сторон также равны.
Доказать, что две равных стороны этого четырёхугольника будут видны из точки, равноудалённой от концов одной диагонали на одно расстояние и от концов другой- на другое, видны под прямым углом.
8 сентября
№42.
Условие: вписанный четырёхугольник ABCD; из B и C опущены перпендикуляры на диагонали AC и BD- BE и CF соответственно и на сторону AD- BG и СH соответственно.
Доказать: 1) из того, что прямые  AB, GE и CD пересекаются в одной точке, следует, что AB, HF, CD также пересекаются в одной точке. 2) 1) выполняется тогда и только тогда, когда прямые  AB и CD перпендикулярны.
13 сентября
№43.
Условие: угол A; точка B внутри него; из B проведены лучи, перпендикулярные лучам угла A и пересекающие его соответственно в точках D и E; точка B симметрично отражена относительно лучей, получены точки D’ и E’ соответственно ; D’E’ пересекает лучи в точках F и G в том же порядке; из F и G восстановлены перпендикуляры к лучам угла , пересекающиеся в точке H.
Доказать: A,H,B лежат на одной прямой; гипотеза: прямую D’E’ можно заменить произвольной параллельной прямой.
21 сентября
№44.
Условие: в прямоугольную трапецию ABCD с основаниями BC и AD (BC<AD) и высотой AB можно вписать окружность; пусть её центр- I;  прямая DI пересекает AB в точке E.
Доказать: AE=BC.
27 сентября
№45.
Условие: в равнобедренной трапеции ABCD  с основаниями BC и AD отмечен центр описанной окружности O. Радиус OC параллелен боковой стороне AB. Прямая DO пересекает сторону AB в точке E.
Доказать: OE=BE.
27 сентября
№46.
Условие: в окружности с центром O проведён диаметр AB; на дуге AB взята её середина C; параллельно AB проведена хорда DE, являющаяся стороной квадрата, вписанного в окружность, CA и CB продолжены до пересечения с прямой DE в точках F и G соответственно.
Найти: угол FOG.
28 сентября
№47.
Условие: прямоугольный треугольник ABC с гипотенузой AB и углом B=p/6. Хорды BD и CE, образующие со стороной BC угол p/4. Прямые DE и AC пересекаются в точке F. O-середина AB. Отрезок FO пересекает биссектрису угла A в точке G.
Доказать: AG=AC.
30 сентября
№48.
Условие: прямоугольный треугольник ABC с гипотенузой AC; на гипотенузе AC как на основании построен равнобедренный треугольник ADC, причём градусные меры углов ACB и ADC относятся как 3:2 соответственно. На стороне AC взята точка E, так что AB=AE.
Доказать: отрезки BE и CD параллельны.
30 сентября
№49.
Условие: прямоугольный треугольник ABC с прямым углом B, у которого отношение катетов AB:BC равно 2:1. На BC как на стороне построен квадрат BDEC, точка E отражена симметрией относительно гипотенузы AC, получена точка E’. Катет BC равен a. Центр вписанной окружности треугольника I.
Найти: расстояние IE’.
30 сентября
Октябрь 2012
№50.
Условие: для равнобедренного треугольника ABC с основанием AC построена вневписанная окружность, касающаяся прямых AB и AC на продолжении соответствующих им сторон. Точка касания со стороной BC- D; CC1-высота треугольника.  Известно, что CD=CC1.  Пусть CC1=h.
Доказать: радиус вписанной окружности треугольника ABC больше величины h/4.
4 октября
№51.
Условие: на прямой AC, содержащей основание равнобедренного треугольника ABC, выбрана точка E (на продолжении стороны AC за точку C), так что отрезок CE равен высоте треугольника CC1. Высота BB1 пересекает отрезок EC_1 в точке F.
Доказать: BC1=BF.
4 октября
№52.
Условие: дан прямоугольный треугольник ABC с прямым углом B. На гипотенузе AC взята середина D, на катетах AB и BC взяты точки E и F, такие, что AE=DE и DF=FC.  Середина отрезка EF –G.
Доказать: AG=GC.
5 октября
№53.
Условие: дан равнобедренный треугольник ABC с основанием AC. Вневписанная окружность касается продолжений сторон AC и AB в точках E и F соотвественно.
Прямая BJ, соединяющая вершину B треугольника с центром этой окружности, пересекает отрезок EF в точке H.
Доказать: BE=BH.
5 октября
№54.
Условие: прямоугольный треугольник ABC с гипотенузой AC. Середины сторон  BC, AC и BC- D, E и F соответственно. Медиана AD. Точка E отражена симметрично относительно этой медианы, получена точка E’. Доказать: отрезок FE’ параллелен AD.
6 октября

№55. Задача на построение.
Условие: вокруг квадрата описана окружность с центром O, середины дуг AB и BC- E и F соответственно, проведены отрезки CE и CF, пересекающие стороны AB и  BC в точках G и H соответственно.
Построить одной линейкой диаметр KL, так, что прямые KG и LH пересекаются на окружности, описанной около квадрата, причём KL не проходит ни через G, ни через H.
8 октября, утро
№56.
Условие: на стороне AC треугольника ABC отмечена произвольная точка D, и около треугольников  ABD и CBD  описаны окружности с центрами O_1 и O_2 соответственно.
Доказать: прямые AO_1 и CO_2 пересекаются на описанной окружности треугольника ABC.
11 октября, утро
№57.
Условие: прямоугольные треугольники ACB и DCE с прямым углом C и точками C, B, E на одной прямой. Окружность, описанная около треугольника DCE, пересекает AB в его середине F. Прямая DB пересекает окружность в точке H.
Доказать: углы FHE и ABE равны.
13 октября, вечер
№58.
Условие: две окружности с центрами O_1 и O_2, касающиеся внешним образом. Проведены общие касательные AB и DE. Известно, что точка пересечения диагоналей C четырёхугольника O_1ABO_2 лежит на окружности с центром O_1.
Найти: отношение радиусов окружностей. 18 октября, библиотека ГПИБ, вечер после 18:00 (до 20:00).

№59.
Условие: равнобедренная трапеция ABCD с основаниями BC (меньшее в общем случае) и AD с точкой пересечения диагоналей O, вокруг  треугольника BOC   описана окружность, пересекающая AB в точке E. Прямая CE пересекает прямую AD в точке H. Угол BAD равен a.
Доказать: площадь треугольника HAE равна ;*AD*HE*sina.

Ноябрь 2012.
№60. (написана в соавторстве).
Условие: дан прямоугольный треугльник ABC. На катете AB во внешнюю сторону построен равносторонний треугольник ADB, а на гипотенузе AC во внутреннюю сторону—равносторонний треугольник AEC. Прямые DE и AB пересекаются в точке M. Весь чертёж стёрли, оставив только точки A и B.
Восстановите точку M.
8 ноября   
№61.
Условие: вокруг треугольника ABC описана окружность, и в ней проведён диаметр AD; высота BE, из B проведён перпендикуляр BF на AD.
Доказать: отрезок EF проходит через середину стороны BC.
8 ноября 2012, вечер
№62. Задача на построение.
Условие: четырёхугольник ABCD с прямыми углами B и D, причём CD>BC.
Построить одним прямым углом точку E на стороне CD, такую, что BE=ED, не ставя ни одной точки вне четырёхугольника.
9 ноября, день
№63. Задача на построение.
Условие: прямоугольный треугольник ABC с прямым углом B.
Построить прямоугольный треугольник DBE, такой, что прямая, содержащая высоту BF первого треугольника, проходит через середину гипотенузы DE – O, а катеты его лежат на прямых, содержащих катеты первого треугольника.
12 ноября
№64.
Условие: равнобедренный треугольник ABC с углом C, равным 2p/3. Высота CE. Прямая CE, на ней выбрана точка D, так, что EC=CD. На отрезке AC выбрана его середина F.
Доказать: BF=BD.
В ночь с 19 по 20 ноября
№65.
Условие: треугольник ABC, вокруг него описана окружность с центром O, в точке B проведена касательная к ней, из точки C проведён перпендикуляр CD на касательную, D отражена симметрично стороны BC и получена точка E.
Доказать: C,O,E лежат на одной прямой.
20 ноября, вечер.
№66.Олимпиада.
Условие: равнобедренный треугольник ABC с основанием AB, вокруг треугольника описана окружность, к ней в точке B проведена касательная, на неё опущен перпендикуляр CD из точки C, проведены высоты AE и BF.
Доказать: D, E и F лежат на одной прямой.
20 ноября
№67.
Определите в прошлом задании, в каком отношении делит точка C диаметр окружности.
                21 ноября (ответ:  1:2)
Квадрат и равенство углов
№68.
Условие: квадрат ABCD, на прямых AD и BD выбраны точки E и F, так, что F лежит в той же полуплоскости относительно AD, что и вершины квадрата AB, и треугольник  EFA- равнобедренный. Прямая CE пересекает прямые BD и AB в точках G и H  соответственно, прямая BC пересекает отрезок AF в точке I. Около треугольника EKD описана окружность, пересекающая диагональ BD в точке J.
Доказать: углы JCD и IGB равны.
29 ноября 2012
Декабрь 2012
№69.
Условие: вокруг квадрата описана окружность с центром O, проведены диагонали AC и BD, на дуге CD выбрана точка G, касательная к окружности в точке G пересекает прямые AC и BD в точках H и E соответствено, из E проведена вторая касательная к окружности EF, BF пересекает AC в точке I, HD пересекает окружность в точке K, прямые  BK  и IG пересекаются в точке L.
Доказать: углы DAF и BLI равны. (н)
№70.
Условие: вокруг квадрата ABCD описана окружность. На дуге BC взята точка E, из D опущен перпендикуляр DF на прямую EC, он пересекает окружность в точке G, отрезки ED и AG пересекаются в точке H.
Доказать: углы HOD,EBC и HFG равны.
№71.
Условие: вокруг квадрата ABCD описана окружность, на меньшей дуге  BC взята точка E, прямые b и DC пересекатся в точке . На отрезке OF взята точка G , такая, что углы  BGD и EBA равны.
Доказать:  углы BGD и EGC равны.


№72.
Условие: вокруг квадрата ABCD описана окружность с центром O проведены диагонали AC и BD.  На меньшей дуге BC взята точка E. Отрезок DE и сторона квадрата BC пересекаются в точке F. Прямая, параллельная AB и проходящая через точку F, пересекает сторону AD  в точке G. На отрезке AG как на стороне построен квадрат AHKG во внутреннюю сторону, прямая GH пересекает дугу AB в точке L. Отрезки EL и BC пересекаются в точке M. Сравнить углы BOM и ADL.
№73.
Условие: вокруг квадрата ABCD описана окружность с центром O, проведены отрезки BO и CO, на меньшей дуге BC взята точкаE, прямые BE и CD пересекаются в точке F, на отрезке OF взята точка G, так что углы BGD и EGC равны. Прямая CG пересекает сторону квадрату AB в точке H, отрезок HE  и сторона BC пересекаются в точке I.
Доказать: OI, AE и BG пересекаются в одной точке.
№74.
Условие: вокруг равнобедренного треугольника ABC с основанием AC описана окружность. Прямая, параллельная AC, пересекает боковые стороны треугольника AB и BC  в точках G и D
соответственно, прямая+ AD пересекает окружность в точке E, назовём касательную к окружности в точке C как l.
Доказать: прямые l, GD и BE пересекаются в одной точке.
№75.
Условие: на стороне AD квадрата ABCD во внутреннюю сторону построен равнобедренный треугольник AED, являющийся тупоугольным. Диаметр  круга AF, на CD выбрана точка G, такая что DF=CG.
Доказать: угол BGE меньше половине угла AED.
№76.
Условие: вокруг тупоугольного равнобедренного треугольника ABC с основанием AC описана окружность с центром O. Около треугольников AOB и BOC описаны окружности и проведена их внешняя общая касательная DE.
Доказать: прямая DO перпендикулярна отрезку BE.
№77.
Условие: в квадрате ABCD проведена диагональ BC. Проведена биссектриса треугольника ACB-CE. На CD выбрана точка F так, что CE=EF, прямые EF и AD пересекаются в точке G, ортоцентр треугольника CEF-P, прямая GP пересекает BC в точке K.
Доказать: BE=BK.
№78.
Условие: трапеция ABCD  с основаниями AC и BD. Стороны AB и CD продолжены до пересечения в точке E, диагонали AC и BD пересекаются в точке O, на отрезках EA и ED взяты их середины F и G. Прямая EO пересекает основания трапеции ABCD-- AC и BD-- в точках I и J соответственно. Прямые JF и GI пересекаются в точке K. Прямая же JF пересекает AC в точке M, а отрезки MG и EJ пересекаются в точке L.
Доказать: отрезки FO,KL и MI пересекаются в одной точке.


Предметный  указатель.
Геометрические места точек—2010, №9; 2011,№13.
Геометрические неравенства—2011,№27.
Гомотетия—2011,№43.
Замечательное свойство трапеции—2012,№78.
Классические задачи на построение—2012, №№13
Комбинаторная геометрия—2011,№33, 2012,№2.
Конкуррентность высот—2011,№№17,19,47.
Необычные построения—2010, №№7,20; 2011, №№16; 2012, №№57,76.
Осевая симметрия—2010, №8; 2011, №№5,27,43.
Параллельный перенос—2011, №17.
Перегруппировка площадей—2011,№№26,32,36,37.
Площадь треугольника—2011,№26.
Площадь четырёхугольника—2011,№26.
Полуправильные многоугольники—2011,№№5,21.
Признаки и свойства параллельности прямых—2010, №№8,11,16,19; 2011, №№18,22,25,46,50.
Признаки подобия треугольников: 2011,№№11,24,28,41.
Признаки равенства треугольников: 2010, №№3,15,19; 2011,№№1,48.
Пятый постулат Евклида—2011,№№6,47.
Свойство касательных к окружности из одной точки—2011,№№30,32.
Свойство ломаной линии--№39.
Свойство медианы из вершины прямого угла—2010, №№10; 2011, №2; 2012, №11.
Свойства параллелограмма—2011,№№6,10.
Свойства прямоугольника—2011, №№20,27,30,31.
Свойства равнобедренной трапеции—2011, №14,16,22,23.
Свойства ромба—2011, №№7,16.
Средняя линия треугольника—2011,№47.
Степень точки относительно окружности: 2010, №1; 2011,№№28,32,50; 2012, №№52,59 .
Теорема косинусов: 2010, №№1,2,6; 2011, №№38
Теорема синусов—2011,№№38,43.
Теорема Менелая: 2012, №78.
Теорема о вписанном угле: 2010, №№2, 8, 17, 18, 21;
2012, №№3,4,5,7,8,9,10,11,12,17,19,20,21,22,71,72
Теорема о смежном угле—2010, №17; 2011,№38.
Теорема о сумме углов в треугольнике и теорема о внешнем угле: 2010, №№10, 12, 13, 14,16,17; 2011, №№4,7,8,22,33,34,44,52; 2012, №№2,8,15,22,48,51, 70,71.
Теорема Наполеона—2011,№34.
Теорема Пифагора: 2010, №№1,4,5; 2011 №№36,37; 2012 №49.
Теорема Птолемея: 2011, №32.
Теорема синусов (усиленная): 2012, №35
Угол падения луча света на поверхность равен углу отражения—2010,№21.
Центральная симметрия--


№1. Подсказка: для нахождения второй величины примените теорему о квадрате касательной.
1) Проведём OB.OD перпендикулярен AC по условию так же, как и OB- AB. Тогда угол BOD=180-угол CAB, поэтому cos BOD= -cos BAC. Дважды применив теорему косинусов, получаем  равенства:  cos BAC=(AB*AB + AC*AC – BC*BC)/2*AB*BC=23/27 и 2AB*AB*(1-cos BAC)=2OD*OD*(1+cos BAC), откуда OD=scrt[AB*AB*(1-cos BAC)/(1+cos BAC)]=1,2scrt2
2) По теореме о квадрате касательной, CD*CD=CE*BC; CD=AC-AD=3 (AB=AD по т. о касательных); BC=5 по условию; значит, CE=9/5.
3) Проведём перпендикуляр OF; по свойству диаметра, перпендикулярного хорде, BF=EF. BF=BC-CE=16/5, BF=EF=8/5.
4) По теореме Пифагора, OF=0,4scrt2.

Ответ: 1,2scrt2 и 0,4scrt2.
 
№2.
Решение: для нахождения расстояния CD достаточно рассмотреть прямоугольный треугольник CDA. В нём уже известна гипотенуза AC (это сторона треугольника). Осталось найти sinCAD. Для второго отрезка решение аналогично.
2010
1) Нахождение sin CA D. Заметим, что угол CBA- тупой, поэтому основание D перпендикуляра CD к прямой лежит на луче AE (E лежит в одной полуплоскости относительно AC, что и B). По теореме об угле между хордой и касательной. Угол CAG (AG-пополнительный луч для луча AE) равен углу CBA. Следовательно, sin CAG=sin CAE(CAD)=sin CBA.
Найдём sin CBA: по теореме косинусов из треугольника ABC находим cos CBA= -1/2.
sin CBA=scrt3/2=sin CAD.
2)CD=AC*sin CAD=7scrt3/2.
3) Проведя аналогичные действия для другого отрезка, получаем, что BE=9scrt3/14.
Ответ: 7scrt3/2 и 9scrt3/14.
 
№3.
Подсказка: докажите, что AD и CF равны.

Решение:
Докажем, что AD=CF.
1) Проведём диагональ квадрата CF. Поскольку угол между диагональю и стороной квадрата равен 45 градусам, то дуги AE и CE, на которые опираются вписанные углы EBC и ABE, равны. Следовательно, AE=CE.
2) Рассмотрим треугольники ADE и CFE. Они- прямоугольные по условию, у них равны катеты DE и CF (стороны квадрата, заданного в условии), а также гипотенузы AE и CE. Следовательно, данные треугольники равны. Значит, AD=CF.
Найдём теперь сторону квадрата
3) Составляем уравнение, обозначив AD за x: b-x=a+x, x=(b-a)/2. Сторона квадрата BF:
BF=a+x=a+(b-a)/2=(a+b)/2.
Ответ: (a+b)/2.
 
№4. Решение:
1) Заметим, что AB=BE как касательные отрезки из одной точки. Пусть BE пересекает сторону CD в точке G, а CF сторону AB- в точке H. Обознасим AD=a, AH=GD=x.
Тогда GH=a, BH=a-x, BC=a+x. Отсюда нетрудно найти x=a/4.
2) Далее, HBCG-прямоугольник, значит, по его свойству, BO=OC=BG/2.
Отсюда находим отношение BO:OE=СO:OF=5:3.
Ответ: 5:3.
 
№5.
Решение:
Обозначим центр окружности I. Тогда AI-биссектриса в треугольнике ABD. Используя свойство биссектрисы и теорему Пифагора, получаем: ID=scrt(a^2+b^2)/2-… Sin BDC=2sinA*cosA=2ab/a^2+b^2. R=ID*sin  ;
№6.
Решение:
Проведём радиусы к точкам касания: OA и OB. Также проведём отрезок OC.
Найдём OC: OC=a/cos 60=2a. Найдём радиус: OA=OC*sin 60=a*scrt3.
 Найдём угол DCO: угол DCO=90-угол ACB/2=30.
Применим к треугольнику DCO теорему косинусов: CD*CD + OC*OC - CD*OC*cosDCO= OD*OD. OD-тоже радиус. Отсюда находим CD=a(scrt2+scrt3).
Ответ: a*(scrt2+scrt3).
 
№7.
Решение:
Для построения достаточно из точки D провести окружность с радиусом DA. Точка пересечения K с прямой BD и будет ортоцентром треугольника. В самом деле, углы KAD и AKD равны 45 градусов в   равнобедренном прямоугольном треугольнике KDA. Угол BKL равен 45 градусов как вертикальный, но угол CBD  равен 45 градусам по условию, следовательно, AL- высота. Мы рассмотрели случай, когда угол ABC-острый. Аналогично рассматривается случай, когда угол ABC-тупой (построение будет абсолютно то же).
 
 
№8.
Решение:
Первое решение: В силу симметрии BK- диаметр окружности, поэтому по свойству
вписанного угла угол BCK- прямой. т.е. прямая CK перпендикулярна стороне BC. Стороны BC и AD параллельны (ромб является параллелограммом), поэтому прямая CK перпендикулярна также стороне  AD, что и требовалось доказать.
Второе решение: диагонали AC и BD перпендикулярны по свойству ромба. Углы ACK(OCK) и ABK равны как вписанные, также равны углы ABD и ADB(EDK) как углы при основании равнобедренного треугольника ABD. Следовательно, равны и углы EDK и OCK, в треугольниках OCK и EDK равны также углы OKC и EKD как вертикальные. Следовательно, угол KED равен углу KOC и составляет 90 градусов, что и требовалось доказать.
 
№9.
Решение:
Заметим, что все  такие касательные отрезки перпендикулярны радиусам окружностей, которых   они  касаются, а эти радиусы все выходят из центра большой окружности, поскольку по условию       окружности концентрические. С другой стороны, все касательные отрезки выходят из точки A, OA- диаметр по отношению ко всем прямым углам, образуемым этими отрезками, следовательно      искомое геометрическое место точек есть окружность, диаметром которой является радиус  большой окружности, проведённый в точку A (исключая центр первой окружности и эту точку).
 
№10.
Решение:
1)Докажем сначала, что угол BID-прямой. Углы GBC и BAD равны как половины равных углов в подобных прямоугольных треугольниках BEC и ABC. Обозначим их a/2. Тогда угол BDA равен 90-a/2, поэтому сумма углов IBD и BDI равна 90 градусов.
Следовательно, угол BID-прямой.
2)Угол JBI равен полусумме углов ABE и EBC, т.е. 45 градусам, следовательно, BI=IJ в равнобедренном прямоугольном треугольнике JIB.
3)Треугольник BAG-равнобедренный, поскольку  у него высота является биссектрисой, поэтому BI=IG.
4)Наконец, медиана EI в прямоугольном треугольнике BEG также равна  BI по свойству прямоугольного треугольника. BI=IJ=IG=IE, ч.т.д.
 
№11.
Решение:
проведём общую касательную к окружностям (внутреннюю) в точке E. Обозначим центры полуокружностей как O и O1. Проведём OO1. Углы при основании треугольника AOE равны, так же, как и углы при основании треугольника EO1D (они равнобедренные по построению).
Углы OAE и EDO1 равны как накрест лежащие при секущей при параллельных прямых a и b.
Следовательно, равны и углы AEO и DEO1, отсюда следует, что они вертикальные и что A, E, D лежат на одной прямой.
Разумеется, прямые AD и BC перпендикулярны, так как соответствующие углы опираются на диаметры.
 
№12.
Решение:
1)Пусть данная трапеция- ABCD, точка пересечения диагоналей-O. Известно, что (AC+BD)<P(ABCD). Тогда поскольку (AB+CD)=(BC+AD), то P(ABCD)=2(BC+AD) (основания) и (AC+BD)<2(BC+AD).
2) Предположим, что BO>BC. Заметим, что если BO>BC, то и AO>AD: угол BOC- наименьший в треугольнике BOC, он равен углу AOD как вертикальный. Углы OBC,OCB,OAD и ODA все равны между собой, так как ABCD-равнобедренная трапеция. Соответственно в треугольнике AOD сторона AD будет по этому предположению наименьшая (т.к. она лежит против наименьшего угла), и поэтому две другие его стороны наибольшие. Далее: поскольку BO=OC и AO=OD, то BO+OC+AO+OD=2(BO+AO)>2(BC+AD). Однако это значит, что AC+BD>P(ABCD), что недопустимо. Следовательно, это предположение неверно, и BO<BC (случай равенства опровергается аналогично).
3) Рассмотрим треугольник BOC. Выяснено, что BO<BC и OC<BC как равный ему. Тогда угол BOC- наибольший. Обозначим его a. Из равнобедренного треугольника BOC можно найти, что угол OBC равен (180-a)/2. Тогда верно неравенство a>(180-a)/2, откуда a>60, ч.т.д.
 
 
№13.
Решение:
1) Углы AMD и BND-прямые, как опирающиеся на диаметры. Угол MDN-прямой по известной лемме (угол между отрезками, соединяющими точку касания двух окружностей, касающихся друг друга внешним образом, с точками касания их внешней общей касательной прямой).
2) Следовательно, в четырёхугольнике MCND угол C-прямой, значит, C лежит на большой окружности. Осталось показать, что CD перпендикулярен AB. Из соотношения углов получаем:  DCN=DMN=CAD=90-MDA=NDB=90-DBN, отсюда получаем утверждение.
 



№14.
Решение: Рассмотрим треугольник AFD. У него известны два угла: угол FAD=45 градусам и угол ADF=60 градусам. Отсюда находим угол AFD=180-45-60=75=углу EFC.
С другой стороны, треугольник EDC- равнобедренный по условию ( стороны правильного треугольника и квадрата равны, так как одна сторона у них общая). Угол EDC=90-угол FDA(EDA)=30. Значит, угол ECD=углу DEC=75. Следовательно, угол EFC=углу  DEC=углу CEF, поэтому CE=CF, что и требовалось доказать.
 
№15.
Решение:
1)Докажем сначала, что прямая AE перпендикулярна BF. Углы CBF и BAE равны, также как и углы BFC и AEB из равенства треугольников FBA и FCB. Следовательно, сумма углов CBF  и AEB равна 90 градусов, поэтому данные прямые перпендикулярны.
2)Итак, треугольник BGE прямоугольный. Докажем, что и треугольник EHC-прямоугольный.
Угол AHC- прямой, как опирающийся на диаметр AC, значит угол EHC-тоже прямой.
 В таком случае у них есть равные гипотенузы (BE=EC по условию) и равные вертикальные углы, а следовательно, они равны, катеты GE и EH  лежат против равных углов, поэтому GE=EH, ч.т.д.
 
№16.
Решение:
Обозначим острый угол ;. Тогда тупой угол по свойству трапеции будет равен 180-;.
Пусть данная трапеция-ABCD с основаниями BC и AD.(BC<AD), точка пересечения данных перпендикуляров-O. Треугольник BOC, очевидно, равнобедренный. Тогда углы при основании треугольника BOC будут равны ((180-;)-90)=90-;, а угол BOC=180-2*(90-;)=2;, что и требовалось доказать.
 
№17.
Решение:
Пусть центр малой окружности- O.
Обозначим угол EAB как a. Тогда угол OAC равен a как угол при основании равнобедренного треугольника OAC. Заметим, что угол BOC равен 90 градусов, т.к. BC-касательная. Следовательно, угол ECB равен 90-a. Однако угол AEB равен 90 градусов как опирающийся на диаметр AB. Следовательно, угол EBD равен a, откуда следует, что дуги, на которые опираются углы EBD и EAB, равны, ч.т.д.
 
№18.
Решение:
Проведём прямую BD и отрезок BO. Угол CBD равен углу DAC как вписанный и составляет 45/2. Угол OBC также составляет 45/2, т.к. BO-биссектриса. OE перпендикулярен BE по условию, значит, треугольник OBF-равнобедренный. Поэтому OE=EF. Но угол ADB=90 как опирающийся на диаметр. Следовательно, треугольник ODF-прямоугольный. Медиана из вершины D  DE=;;=EF, откуда следует равенство.
 

№19.
Решение:
Продолжим стороны AB и AC за вершины B и С соответственно. Проведём все перпендикуляры из точки D к сторонам треугольника или их продолжениям: DE к AB,
DF к BC, DG к AC.  Тогда угол DBC как накрест лежащий при секущей BC равен углу ACB, а угол DBE  равен углу CAB  как соответственный при секущей AE и также равен углу ACB.  Тогда углы  DBE и DBF равны  Следовательно, треугольники EBD и FBD равны по гипотенузе и острому углу, т.е. DE=DF. С другой стороны, углы BDC и BCD равны, т.к. треугольник CBD – равнобедренный. Также угол BDC равен углу DCG как накрест лежащий при секущей CD. Поэтому углы BCD и BCG равны.
Поэтому аналогично DF=DG, но это и значит, что D является центром вневписанной окружности треугольника.
 

№20.
Решение:
1) Пусть данные квадраты- ABCD и DEFG. Проведём линейкой прямые AC и EG.  Пусть их пересечение - точка H.  Две стороны квадрата уже построены: рассмотрим треугольник AHG. У него  углы HAG и AGH равны 45 градусам ( поскольку прямые AC и GH содержат диагонали), следовательно, треугольник AHG- прямоугольный  и равнобедренный.
2) Продолжим BC до пересечения со стороной FG в точке I. Проведём диагонали  AI и BG и обозначим их точку пересечения J.  По свойствам симметрии HО пересечёт отрезок AG в его середине K.
3) Проведём луч BK и продолжим FG; обозначим их точку пересечения L. Из равенства треугольников BAK и KJL (по катету и острому углу) следует, что отрезок GL равен AB.
4) Проведём луч FD и продолжим BA; обозначим их точку пересечения M. Так как угол FDG равен углу ADM и равен 45 градусам, а угол A- прямой, AM=AD=AB.
5) Проведём  отрезок  ML и продолжим ED; обозначим их точку пересечения N.  Ясно, что квадрат ABCD симметричен квадрату AMND, поэтому прямая AN содержит сторону искомого квадрата.
6) Проводим прямые AN и  HJ; обозначим их точку пересечения O.
7) Строим GO. В силу симметрии (AG и HO- оси симметрии) AHGO- искомый квадрат.
 
№21.
Решение:
Проведём отрезки BE и CE. Дуга BC равна 90 градусов, поскольку стягивает четверть окружности. Следовательно, угол BEC=45 как вписанный (т. о вписанном угле). Далее, угол BED равен 90 градусов как опирающийся на диаметр.
Так что угол DEF равен 45 градусам=углу BEC, откуда следует утверждение задачи.

2011
№1. Решение:
1)Заметим, что треугольник AOD- равнобедренный (возьмём это как лемму; лемма следует из того, что трапеция ABCD-равнобедренная). Поскольку AO1 и O1D- биссектрисы равных углов при основании этого треугольника, то AO1=O1D.
2)Треугольники CAO1 и DAO1 равны по двум сторонам и углу (AC=AD по условию, углы CAO1 и DAO1 равны, AO1-общая сторона). Следовательно, DO1=CO1.
3)Ясно, что CO1=BO1. Следовательно, O1 есть центр описанной окружности трапеции, что и требовалось доказать.

 

№2. Меньшее основание BC трапеции ABCD относится к большему основанию AD как 1:2. В трапеции проведена высота DE к боковой стороне AB.
Доказать: CE=CD
Первое решение.
Продолжим AB и DC до их пересечения в точке E. Тогда BC- средняя линия треугольника  AED, так как обладает всеми свойствами, ей присущими (равна половине основания и параллельна ему).Тогда по свойству медианы из вершины прямого угла CE=CD.

 
Второе решение
1)Возьмём на большем основании AD точку F, такую, что AF=FD. Тогда AF=AD-FD=AD/2=BC.
2) Поскольку BC=AF и BC параллелен AF, то, по известной лемме,ABCF- параллелограмм, и AB параллелен CF. По свойству параллельности, CF  перпендикулярен CE. По теореме Фалеса, DG=GE.
3) По признаку равнобедренности, так как высота CG является в то же время медианой, треугольник ECD- равнобедренный, CE=CD, ч.т.д.
 
№3.
Решение:
Докажем сначала, что BD-биссектриса угла EBC. Углы OBE и OAE равны как вписанные, а углы OAE и CBO равны по свойствам равнобедренной трапеции. Следовательно, равны и углы OBE и CBO. Следовательно, расстояния от точки O, принадлежащей биссектрисе, до прямых BC и BE равны. Нетрудно заметить, что OH (расстояние до BC) равно разности отрезков HF и OF, HF является высотой трапеции, поэтому и расстояние от ;L до BE равно разности высот BG и OF.
 
№4.
Решение:

Пусть пересечение прямой AF и стороны CD- G. По свойству равнобедренного треугольника и по свойству симметрии получаем: угол FBA=90-CBF=90-FDC=GFD=BFA. Отсюда утверждение задачи.
№5.
Решение:
Обозначим в задаче точку пересечения второй окружности со стороной AB как H. Тогда по всем известной теоремке о точке, симм. точке пересечения высот относительно стороны треугольника, BH- диаметр этой окружности. Значит , центр второй окружности ; лежит на AB. Для утверждения задачи достаточно доказать, что OG перпендикулярен AE. Углы OGB и OBG  равны, как и углы OBG и GBE, следовательно, равны и углы OGB и GBD, т.е. OG параллелен BE, откуда непосредственно следует утверждение задачи.
 
№6.
Решение: DG=GC по теореме Фалеса. Пусть точка пересечения EH с окружностью- I; EG=GI (известная теорема. Тогда прямая DI параллельна AC, а значит, совпадает с FD, также ему параллельной (пятый постулат Евклида).
 
№7.
 Указание:
Надо выразить углы треугольника DEF через углы треугольника ABC  и доказать, что треугольник KLM имеет те же углы, что и ABC, после чего воспользоваться свойством ортотреугольника: высота треугольника, в который он вписан, является его биссектрисой.
 
№8. Решение:
Обозначим точку касания EF с окружностью как M.
Проведём CM и CF (CE уже проведён). Тогда нетрудно заметить, что СМ=CB=CD, и поэтому CE и CF- биссектрисы соответственно углов DCM и BCM, однако сумма последних равна 90 градусов, следовательно, угол ECF равен 45 градусам и равен углу EDG. Тогда легко заметить, что в таком случае точки D, E, C, G лежат на одной окружности и потому угол FGC-прямой (так как прямой и угол FDC по условию). Отсюда вычисляется, что угол GFC равен 45 градусам, т.е. FG=CG. C помощью аналогичных рассуждений можно получить, что  EG=HG. Значит, EG+HG=CG+GF, CF=EH, ч.т.д.
 
№9.
Решение:
Обозначим пересечение прямой BE со стороной CD квадрата как M.
 Заметим, что угол AED- прямой, как опирающийся на диаметр. Следовательно, точки A, E, L, B лежат на одной окружности. Из последнего следует, что углы BAL и BEL равны. Углы BEL и MED равны как вертикальные, углы MED и EAD как вписанные, также равны углы MDE и MED. Итак, все эти углы равны между собой, поэтому в силу равенства сторон квадрата прямоугольные треугольники KAD, BAL и LDC равны по катету и острому углу, из чего можно заключить, что LK- средняя линия треугольника CBD и поэтому параллельна диагонали BD, ч.т.д.

№10
Решение: пусть точка касания окружностей B. Проведём через неё прямую, содержащую диаметры окружностей и обозначим их необщие концы как A и C. Пусть искомый отрезок- DE. Проведём AD, BD, BE и CE и рассмотрим треугольники ADB и BEC. У  них углы D и E прямые, гипотенузы AB и BC равны по условию, и, кроме того, острые углы: BD параллелен EC , т
ак как углы DEC  и DBE оба равны 90 градусов (признак параллельности прямых).   Так как BD параллелен и равен EC, BDEC- параллелограмм, поэтому DE  равен и параллелен BC, ч.т.д.
Для второго возможного отрезка решение аналогично (см. рис.)

 

№11.
Решение:
Легко доказать, что LN=1/5 ND. Опустим из L перпендикуляр на BC- LP. PN=1/10 BC Пусть S- точка пересечения KL с BC. Тогда PS=PL=1/5BC, BS=1/5 BC ( KL параллелен AC по разным свойствам). И далее ясно.
 
№12.
 Решение: по теореме о вписанном угле  угол BDC равен углу DAB (DAC); так как ADC- равнобедренный, углы DAC и DCA (DCB) равны; тогда  углы DCB и DBC равны. Следовательно, BD=BC. Но  AD перпендикулярен CD. Тогда по свойству медианы из вершины прямого угла BE=BC
 
№13.
Решение: Пусть O1 –центр описанной окружности треугольника OAC, O2- треугольника OAB. т.к. O1;=;1; и O2O=O2A, O1O2 перпендикулярен ;A и делит этот радиус пополам.  Ясно, что для остальных треугольников выполняется то же, поэтому O равноудалён от центров описанных окружностей на расстояние R/2, где R-радиус описанной окружности треугольника ABC.
 
№14.
Доказательство формулы z=R(x-y)/b, где  z-расстояние между центром описанной окружности и точкой пересечения диагоналей, x и y -отрезки, на которые  их делит их точка пересечения, соответственно больший и меньший, R- радиус описанной окружности трапеции , b -боковая сторона трапеции.
Пусть точка пересечения диагоналей трапеции-K, основания- BC и AD. Рассмотрим угол ADB. Он равен как вписанный половине угла AOB, а угол AKB как внешний при боковой стороне равнобедренного треугольника равен удвоенному углу ADB, из чего можно видеть, что углы AKB и AOB равны. Поэтому A,O,K,B лежат на одной окружности, следовательно, четырёхугольник AOKB-вписанный , следовательно, для него применима теорема Птолемея. AK=x,BK=y, OB=OA=R, OK=z, AB=b. Составляем уравнение, пользуясь обозначениями: Rx=Ry+bz, откуда z=R(x-y)/b.
Из этой формулы была выведена ещё одна:
z=(x^2-y^2)/2h, где z-расстояние между центром описанной окружности равнобедренной трапеции и точкой пересечения её диагоналей, x и y- отрезки, на которые она их разбивает, h-высота трапеции. Доказательство: Пусть a-тупой угол трапеции. Тогда (x+y)=2Rsina, R=(x+y)/2sina. По предыдущей формуле z=R(x-y)/b=>z=(x-y)(x+y)/2sina*b. sina=sin(180-a)=>sina*b=h=>z=(x^2-y^2)2h.
 

№15. Решение:
Пусть данный треугольник- ABC и D и E- середины соответственно сторон AC и BC. Проведём BD и AE. Поскольку треугольник равносторонний, проведённые отрезки будут не только медианами, но и высотами. Кроме того, поскольку через их точку пересечения H проходит также высота и медиана, F- середина AB.  По теореме Фалеса (поскольку EF параллелен AC) BG=GD.  Также, поскольку ABC- равносторонний, FG=GE. Проведёv прямые AG и DE: докажем, что точка их пересечения принадлежит прямой, параллельной AC.  Поскольку треугольники GBI и ADG равны по катету и острому углу, их гипотенузы AG и GI; равны. Тогда ABI;D- параллелограмм. Но DE параллелен AB как средняя линия, поэтому по аксиоме Евклида ясно, что DI параллелен AB и BI параллелен AC.  Проведём теперь прямые BI и AE и обозначим точку пересечения как J.  Так как Углы CBA, СBJ оба равны 60 градусам, треугольгики ABE и JBE равны по катету и острому углу.  Тогда AB=BJ=BC, но BJ параллелен AC, и тогда по установленному мной факту для равнобедренных треугольников получим, что J-центр вневписанной окружности.
Замечание: ясно, что совершенно так же можно построить два остальных центра вневпиванных окружностей.
 
№16.
Решение:
Радиус OB параллелен CD.=> углы OBD и BDC равны. Также равны углы OBD и ODB. Пусть OE-искомое расстояние; Тогда DB-биссектриса угла ODC. Следовательно  (продолжим прямую OE до пересечения с CD в точке F),OE=EF. Далее: так как OBFD-ромб,BF=OC. Так как в трапеции OBCF равны диагонали, OF=BC=2OE, отсюда следует утверждение задачи.

 
№17.
Как несложно доказать, BE=BF, а так как FE; параллельна AB, то она перпендикулярна BC, откуда BE;перпендикулярен CE, дальше ясно.

 
№18.
Решение:
Достаточно заметить, что углы EBD=EDA по теореме о вписанном угле и углы FED=DBF, поэтому FED=EDA, т.е. EF параллелен AC.

 
№19.
Решение: так как углы GFE и GAE равны как вписанные, углы GAE и DAE равны по условию, то углы GFE и DAE равны, поэтому отрезок EG перпендикулярен AE, аналогично отрезок EH перпендикулярен AF.
 
№20.
Решение: Докажем сначала, что AF=EF. Углы DBA и ACD равны как вписанные и опирающиеся на одну дугу; углы CAF и ACD равны как накрест лежащие при параллельных прямых AF  и CD. Отсюда равенство углов ABF и FAB. Угол ABC-прямой, поэтому AF=EF. Тогда AG=EG по признаку равнобедренного треугольника.
Далее: AFGD-прямоугольник (три прямых угла). Тогда по разным его свойствам равны углы GDF и GAF. Также равны углы CDB (GDF)  и CAB (как вписанные). Тогда нетрудно заметить, что равны углы CAF и CAG. Отсюда GC=GA.
 


№21
Решение: пусть данный шестиугольник-ABCDEF. Поскольку он равноугольный, все его углы равны 60 градусам. Рассмотрим четырёхугольник ABCD. Поскольку он вписанный (его вершины лежат на окружности, описанной около шестиугольника, а он будет всегда вписанный), угол CDA равен 60 градусов. Тогда угол ADE также равен 60 градусам, поскольку CDE равен CDA+ADE Если применить «утверждение balu» (//http:geom.uz.) и заметить, что AD=BE=CF,нетрудно доказать и второе утверждение.





 
№22. Решение: пусть данная трапеция- ABCD, центр описанной окружности- O. Пусть BO параллелен CD. Докажем, что угол AOD=3 угла BOC. Обозначим угол BOC как a. Углы при основаниях равнобедренных треугольников COD и AOB будут также равны a (по свойствам параллельности и равнобедренного треугольника. Углы при их общей вершине O будут равны 180-2a, следовательно, угол AOD=360-a-2*(180-2a)=3a.
Обратное утверждение доказывается сходным образом. (угол ABO находится из уравнения:
(180-3a)/2+(180-a)/2+2x=180)


 



№23.
Указание: найдите углы трапеции и углы между её диагоналями и сторонами и воспользуйтесь предыдущим утверждением.

 
№24. Решение: Заметим, что прямая EF перпендикулярна биссектрисе BD (выведите это, применив свойство биссектрисы угла).Обозначим их точку пересечения как H Тогда по свойству прямоугольного треугольника треугольник AHD- равнобедренный. Тогда углы AHB и CDB равны как пополнительные равным углам. Отсюда и из условия следует, что треугольники AHD и CDB подобны. Тогда BH/AB=(BH+HD)/BC, откуда BH:HD=a:b. Но треугольник DEB (можно совершенно также рассматривать треугольник DFB)-прямоугольный из условия. Тогда по свойствам подобия нетрудно найти  отношение его катетов ( ведь по доказательству EH-высота). Отсюда находится sinDBC, cosDBC и sinABC=sin(2DBC).
 


№25.
Решение:
Поскольку треугольник ABC- равнобедреняный и AB и AC- касательные, то:  О1СD = 90 - ACB = 90 - ABC = DBO отсюда и следует утверждение задачи (признак параллельности прямых).

 

№26.
Решение: обозначим радиусы R и r, сторону ромба-a, а диагональ (линии центров)-d. Тогда получаем: Ra/2+Ra/2+(R+r)*d/2+ra/2+ra/2=S ромба. Так как S постоянно, указанная сумма постоянна.
 
№27.
Указание:
Для решения достаточно доказать, что радиус вписанной окружности равнобедренного треугольника меньше половины основания.  Последнее легко доказать, если воспользоваться свойством равнобедренного треугольника: углы при основании меньше 90 градусов.
 
№28. Решение: нам известно, что BC^2=CD*AC. Из условия: EC^2=CD*AС=AE^2. Отсюда нетрудно вывести подобие треугольников EDC и AEC. Однако EC=AE. Следовательно, равнобедренный и треугольник EDC, причём равны именно стороны ED и DC, так как против AC лежит больший угол и кроме того EC=/=CD.
 
№29.

По свойствам симметрии получается, что AO содержит высоту. Около четырёхугольника OBCA, очевидно, можно описать окружность, тогда из теоремы о точке, симметричной основанию высоты, получаем утверждение. (FG=GO).
 
№30.
Решение: подсказка: проведите вторую касательную из центра O1 к малой окружности и докажите, что точка её касания с ней лежит на одной прямой с O и M.
Решение: проведём касательную O1L1 из O1 к малой окружности. Обозначим угол KO1O как
a. Тогда нетрудно доказать, что угол MOL (L-точка касания) равен 2a. С другой стороны, угол   LO1L1=2a, тогда угол  LOL1=180-2a, отсюда следует, что MOL+ LOL1=180, но это и значит, что M,O,L1 лежат на одной прямой.
У четырёхугольника L1MNO1 три прямых угла, следовательно, он является прямоугольником, поэтому O1N=ML1=2MO.
 
№31.
Решение. OECO1 – прямоугольник, так как OE=O1C, и O1C  перпендикулярен  AC, как и OE (по свойству диаметра, перпендикулярного хорде, проекция O на AC совпадает с серединой AB). Тогда EO1=OC.
Далее заметим, что поскольку DO1=O1С, и угол DO1C=90, угол CDO1=45. Угол ODC=180-    угол CDO1=135. По формуле, OC/sin135=2R, откуда R=1.
 
Подсказка: вычислите синусы углов между каждым из данных отрезков и стороной BC.

№32. Решение: обозначим центр окружности O, радиусы, проведённые к соседним вершинам данной фигуры как OA и OB, точки касания касательных отрезков, проведённых из B и A соответственно к меньшей окружности- E и F, а точку пересечения их - как D. Тогда ясно, что прямоугольные треугольники AED и BFD равны по катету и острому углу (отрезки BE=AF и являются высотами в равнобедренном треугольнике AOB).  Нетрудно доказать, что AF=FC (С-следующая вершина солнышка, соседняя с B).  Отсюда следует утверждение (если рассмотреть все аналогичные пары треугольников).
 
№33.
Решение: это равнобедренный треугольник с углом 120 градусов. В самом деле, разделим его основание на 3 равные части и проведём из противолежащей вершины две прямые к точкам деления. Тогда по бокам окажутся два равнобедренных треугольника с углами 30 градусов, а посередине-равносторонний треугольник.
 
№34. Решение: пусть данный треугольник- ABC. Рассмотрим его сторону AC. Пусть данные точки деления-A1 и C1, причём A1 лежит ближе к A, а C1- к C. Пусть точка пересечения полуокружностей-E. Заметим, что треугольник A1EC1-равносторонний, поскольку он образован радиусами равных окружностей. Тогда, поскольку AA1=A1E=C1E=A1C1=CC1, углы EAC1 и ECA1 равны как вписанные по 30 градусов. Тогда угол AEC равен 120 градусов, треугольник AEC- равнобедренный. Если теперь построить на AC равносторонний треугольник, E окажется его центром (по свойству вписанного угла, ведь любой угол равностороннего треугольника равен 60 градусам). Тот же ход рассуждения верен и для двух других сторон. Ясно, что если теперь применить теорему Наполеона, получим утверждение задачи.
 
№35. Решение: 1) обозначим  углы O3O1O2 и O3O2O1 как a и b. Рассмотрим треугольники C1O1C2 и O2A1A2. Они равнобедренные и углы при их основании равны соответственно 90-a/2 и 90-b/2.
2) Рассмотрим теперь четырёхугольник O3C2BA2. У него  углы O3C1B и O3A1B составляют как смежные с соответствующими углами величины 90+ a\2 и 90+ b/2 соответственно. Тогда угол  C1BA1 равен по теореме о сумме углов в многоугольнике 180-a/2-b/2, откуда по теореме о сумме углов в треугольнике находится этот угол. Ответ: 60 градусов.






 
№36
 
Ответ: a^2/4-2b^2+ab*scrt2; a^2/4+b^2*(scrt2+2)/2-2ab*(scrt2+1); a^2/4+b^2(scrt2+1)/2-ab*scrt2*(scrt2+1)/2.





№37.
Обозначим ширину рамки b, HF1 как x. Тогда найдём длины сторон треугольника D1E1F1: B1D1=b*scrt2,  высота из точки D1 равна a/2*scrt2-b-b*scrt2, GH=2b, x=a/2*scrt2-2b-b*scrt2; E1F1=a/2*scrt2-2b-2b*scrt2. DD1=a/2*scrt2-2scrt2*b=2KG=x*scrt2.


 
Ответ: b=a(scrt-1)/2scrt2
Ещё раз посчитал 27 июня 2012
№38.
Решение:
Угол BAC<60 как наименьший (против меньшего угла лежит меньшая сторона), угол EAC также – меньше 60 градусов, путём счёта углов получаем, что угол AGB<150, угол же AOC>150, как то можно вывести из теоремы косинусов. Ответ: AOC>AGB.


.
 
№39.
Решение: по теореме о вписанном угле угол OAD равен углу ODC и угол ODA равен углу OAB, следовательно, по свойству параллельных прямых треугольники ABC и BCD- равнобедренные, т.е. все три отрезка равны, далее теорема о ломаной линии.
 
№40.Решение: пусть точка, диаметрально противоположная B-B1. Тогда C,D,B1 лежат на одной прямой (по теореме о вписанном угле и по теореме о единственности перпендикуляра из данной точки). Углы DB1B и DAB равны как вписанные и опирающиеся на одну дугу. Углы CAE и CB1E равны по аналогичной причине. Углы DAB и DBA равны как прилежащие к основанию равнобедренного треугольника. Из всего этого можно заключить , что равны углы BAE(CAE) и DBA, а значит, по признаку параллельности, DB и AE параллельны.

 

№41
Указание: докажите, что у этих четырёхугольников по две пары равных сторон (исходя из подобия с коэффициентом 1). Ответ: параллелограмм, дельтоид.

 


№42.
Решение: по теореме синусов находим AC, по свойству вписанного четырёхугольника находим угол ADC, по теореме косинусов, составив уравнение (где за x принимается одна из сторон AD и CD) получаем две стороны. Две другие находим по теореме Пифагора.
№43. Решение:  1)заметим, что BI=2AE:  BI=AB-AI, AE=(AD-EH)/2, EH=EF=AI.
2) Т.к. EF перпендикулярен IG, то, по признаку равнобедренного треугольника, EI=EJ.
3) FG параллелен  BC, из чего нетрудно вывести, что JG=GK, тогда и EJ=EK.
4) E-центр окружности, описанной около  треугольника IJK, поэтому по теореме о вписанном угле угол JKN=1/2 угла IEJ. Углы  IEN и  JEN равны по свойству равнобедренного треугольника, поэтому углы JKN и JEN равны, следовательно, ENJK-  вписанный.
 

№44. Доказательство: 1) Докажем,что все углы EBO,GBO,CAO  и ACO равны между собой. Для этого достаточно заметить, что так как OB=AO и AE=BE, то, по признаку равнобедренного треугольника углы EBO и CAO равны. Аналогично доказывается и для другой пары углов (очевидно равенство углов CAO и ACO) . 2) Обозначим угол CAO  за a. Угол   HOI=180-2a,угол HBI=2a. Следовательно, B,H,O,I лежат на одной окружности. Поэтому 3) Угол HIO равен углу IBO, поэтому равны и углы IHO и CAO, а следовательно, по признаку параллельности,  прямые HI и AC параллельны.
Комментарий: можно доказать проще: если BO пересекает AC в точке J, то доказывается: OH=OJ=OI, и отсюда выводится утверждение.
 
Комментарий. 13 июня 2012 г. было обнаружено, что J-центр вписанной окружности треугольника BHI (т.к. угол HIJ=(угол HOB)/2=угол HIB/2). В этот же день было обнаружено, что, если продолжить  IJ до пересечения  с  OD  в точке K, это будет центр описанной окружности треугольника  AJB. (первое было обнаружено до 12:00, второе- с 17:00 по  17:15).
№45.Доказательство:  Углы  CAO и ACO равны как углы при основании равнобедренного треугольника. По свойствам равнобедренного треугольника равны  также углы FBO и CAO (FAO). Так как ADEO вписанный (по теореме о вписанном угле: углы ADO и AEO прямые), то равны углы EDG и FBO (FBG). Поэтому FDBG- вписанный, значит, так как угол FDB (EDB)- прямой, то угол FGB- также прямой, следовательно, FG перпендикулярен BO.







№46.
Решение:
Угол CBD=углу  CAD, угол CBE=углу FAE=> угол FAC=углу DBE=>ABHI- вписанный=> угол BAF равен углу BIH. Но угол BAF углу  BEF=>;; параллелен EF, но т.к. EF параллелен CD, HI параллелен CD. Используемые теоремы: о вписанном угле, о накрест лежащих углах и др.
 
№47.
Решение: так как BF=СF, и углы JBF и ICF равны, a BI перпендикулярен OC (конкуррентность высот), BJ=CI и JI || BC. Но JD||BC как средняя линия, поэтому JD и JI совпадают, то есть I,J,D лежат на одной прямой.
 
№48.
Решение. Заметим, что треугольники DGF и IHE равны (I-проекция H на AC); угол DFG=DEB значит EI перпендикулярен BC, откуда I- середина AC. Далее- признак равнобедренного треугольника.
 

№49
Углы KAB (KAD) и KCB равны по свойствам равнобедренного треугольника, углы DCK и KCB равны по условию, значит углы KAD и DCK равны, тогда, по теореме о вписанном угле, A,D,C,K лежат на одной окружности.

 




№50 Известен факт, что прямая, соединяющая точки пересечения двух пересекающихся касательных, делит отрезок их общей внешней касательной, заключённый между точками касания, пополам. Тогда, применив обратное утверждение, получим, что угол EAC равен углу DBA. Тогда угол FEA равен сумме углов DBA и DBC и равен углу FBG, следовательно, E,F,B,G лежат на одной окружности, значит, углы GBE и GFE равны, то есть равны и углы GFE и ECA, значит, прямые FG и AC параллельны (аналогично можно рассмотреть другие пары углов)


 
№51
Обозначим  угол BAC как a. Проведём высоту CH. Аналогичную  DF. Они, по свойству симметрии, пересекутся на высоте BG. Углы FDC и HCA равны 90-a, значит, угол DOC равен 2a по теореме о сумме углов  в треугольнике. Следовательно, так как по условию угол DBE равен a,  угол DOC=2 угла DBE, значит, по теореме о вписанном угле, O-центр описанной окружности треугольника DBE.

 
№ 52
AE=AC, поскольку AD=DE=AB=BC. Значит, углы ACE и AEC равны и составляют 67,5 градусов. Аналогично углы ABD и ADB составляют столько же. Поэтому DFC-равнобедренный,  следовательно, DF- медиана прямоугольного треугольника EDC, откуда и следует утверждение.


 

№1.
Решение: EG параллелен CD, т.к.: EG=AE=BE по свойству медианы из вершины прямого угла => углы EAG и AGE равны; кроме того, трапеция равнобедренная => углы BAG и CDA равны; значит, равны и углы EGA и CDB; из этого следует данный факт.
Т.к. EG=СА и EG параллелен CF, CEGF-параллелограмм. По теореме косинусов, FG>CG. Т.к. CG-наклонная, то CG>BG. Но EF= (BC+AD)/2. Из этого следует утверждение.

 
№2. Решение: пусть AB=BE, BC=CE,BC||AE, BE||AD. Пусть угол CBE=a. Тогда угол BCE равен 180-2a по теореме о сумме углов  в треугольнике. Угол BAE равен углу BEA равен углу EAD равен a по свойству равнобедренного треугольника и по свойству параллельности.. => углы BAD и BCE в сумме составляют 180 градусов, следовательно, четырёхугольник ABCD- вписанный.

 
№3.
Решение: продолжим EF до пересечения с окружностью в точке I. ;;=CD по свойствам симметрии и EI || CD =>ID||EC . Так как CH=CD=EI, H принадлежит ID. Углы EIH и CDH равны как соответственные, углы EBH и EIH- как вписанные, следовательно, DGHB-вписанный. Поскольку же угол BDG- прямой, BH перпендикулярен GH, значит, поскольку AHB- опирающийся на диаметр, A,G и H лежат на одной прямой.
 

№4.
Решение: Из D проведём окружность с радиусом CD, пересекающая окружность в точке G. Отрезок GC продолжим до пересечения с окружностью в искомой точке E.
Доказательство: FD параллелен EG, т.к. EF=DG. Т.к. Углы FEG и DCG равны и равны углы FEG и DGF, CD=DG. Верно и обратное.
Комментарий: вторая возможная точка строится аналогично.
 

№5.
Решение: угол DOD1=углу COC1=углу CEC1 = углу D1C1F=>OC1GD1- вписанный=>D1G перпендикулярен C1F. Но D1-ортоцентр AC1F ( C1D1 перпендикулярен EF по свойствам параллельности, FO перпендикулярен A1C1, так как A1B1C1D1- квадрат) => AG проходит как высота через D1, отсюда утверждение задачи.
 

№6.
Решение: пусть точка пересечения данных касательных- F, точки касания окружности с лучами угла- G и H; так как OB=OC=BF=FC=OG=OH=GA=HA, BD=DG, HE=CE,  то DE-линия симметрии в четырёхугольнике ADFE, отсюда следует утверждение.

 
№7.
Угол FBD=углу FAD и угол CBD=углу CED как вписанные=> угол CED=углу FAD=>AF||CE. (теорема о вписанном угле, признак параллельности прямых). Т.к. AF||CE, углы FAC и ACE равны как накрест лежащие, и равны углы AFG и FDG как вписанные=> D, H, G и С лежат на одной окружности. Углы DGH и HCD равны как вписанные так же, как и углы ECD и EBD (окр.2). Тогда равны и углы DGH и DBJ  (J-пересечение прямой ;; и окр.2). Но B, G, D, J лежат на окр.1, поэтому G,H,J лежат на одной прямой. Угол  IKG= углу  ABJ=углу AGJ=> KI || GH.

 




№8.
1) 1) OD||CA, так как соответственные углы при их секущей AB равны и составляют 45  градусов. По теореме о сумме углов несложно посчитать, что углы ODG и  GCA составляют 33,75 градусов. Это и значит, что O,G,D лежат на одной прямой.
2) Так как отрезки CO и OD перпендикулярны, а угол CED=135 градусов как противолежащий углу 45 градусов (теорема о вписанном угле),OE,D лежат на одной прямой. Тогда угол FEA=45, а так как угол EOF=22,5, то OF=EF.
3) O1C и O1D перпендикулярны, так как по теореме о вписанном угле, угол  CO1D вдвое больше угла  DAC=BAC; с другой стороны, CO и OD также перпендикулярны (что было установлено выше). Тогда  CODO1- вписанный, поэтому угол DOO1=углу DCO1=45, так как угол BOD=45, то угол BOO1 равен 90, поэтому OO1||AB.
 


№9.
Решение:   1) Угол ABF и угол AGF равны как вписанные; углы ABF и DCF равны как острые углы в равных прямоугольных треугольников, лежащие против равных катетов.  Так как  CG перпендикулярен BH,  углы GCH(DCF) и CBH равны. Значит, углы CBH и ABF равны. Но так как BC=CE, BE- биссектриса угла ABC, следовательно, углы  FBE и EBG равны.  Так как углы FED и BEC составляют 45 градусов, угол FEB=90, тогда из теоремы о вписанном угле (применённой к четырёхугольнику ABEF) следует утверждение задачи (1). 2)
Комментарий: 24 мая: GE=FE (рассматривается  четырёхугольник AGEF, являющийся вписанным, так как все углы ABF, AGF, AEF равны между собой).
 
№10.
1) Пусть точка пересечения окружности и прямой EJ-G. Углы CED и ABF равны по теореме о вписанном угле, углы AEB и  CED равны по свойству симметрии, значит, углы CED и ABF равны. С другой стороны, углы AEB и CBE равны как накрест лежащие при секущей при параллельных прямых.  Поэтому углы ABF и CBE равны. Отсюда углы CBL и ABE равны. Также равны углы ABE и AFE по теореме о вписанном угле. Углы CBL и BLA равны как накрест лежащие при секущей при параллельных прямых. Тогда равны углы  BLA и AFE, т.е. AF- касательная к окружности, описанной около EFL. AG-диаметр окружности, описанной около треугольника ABF, так как отрезок FG перпендикулярен AD. Тогда  G,F,K лежат на одной прямой, и, очевидно, J-ортоцентр треугольника AGK, тогда  прямая JK перпендикулярна   AG. Так как углы ABF и AFB равны, AB=AF и AG перпендикулярен BF. Поэтому по признаку параллельности прямых, JK||BF.
 
1) Обозначим угол EFL как a. Тогда угол BAD равен a, так как ABFE- вписанный, так как O-центр описанной окружности EFL, угол TLE равен (90-a), поэтому отрезок LO перпендикулярен AC; углы BAG и GAF равны из симметрии, BLT и BAG равны, так как lT- высота треугольника ABL, а AG содержит в себе высоту этого же треугольника (теорема о вписанном угле); поэтому A,P,F,L лежат на одной окружности, поэтому по теореме о вписанном угле углы PFA и PLA равны, кроме того, угол TQG=EQL=90-QLE=a; а так каотрезок AF перпендикулярен GK, то угол GFP также равен a, отсюда следует утверждение
 
№11.
Решение:  пусть OE пересекает BD в точке L (множество точек, равноудалённых от двух данных;свойство касательных из одной точки); тогда OL перпендикулярен BD; DF-прямая, содержащая высоту (конкуррентность высот); тогда DF параллелен ;; (свойства параллельных прямых); тогда поскольку BC параллелен DE и угол BDB1- прямой (угол, опирающийся на диаметр), BF=FH. Тогда т.к. BG=CG, нетрудно вывести, что CH=2FG.
 


№12.
Решение: проведём отрезок DE. Пусть точка пересечения с ним биссектрисы угла DGE-F, а точка пересечения FG и CD-J. Тогда, поскольку треугольники DGE и DGC-равнобедренные, а углы DCA и DEA равны как вписанные, углы DEA и GEJ равны. Тогда биссектриса угла DEG совпадает с биссектрисой угла HEJ. Пусть K’-центр вписанной окружности треугольника DGE.  Тогда углы HEK’,K’DJ и JDK’ равны, следовательно, по теореме о вписанном угле, D,H,K’,E лежат на одной окружности. Поскольку B,D,H,E также лежат на одной окружности, K лежит на этой окружности. Поскольку треугольник DGE-равнобедренный, по его свойству и по теореме о вписанном угле, углы, углы DEK’,EDK’,K’BD и K’BE равны. Тогда очевидно, что K’ совпадает с K. Поскольку BH-диаметр (поскольку угол HDB по условию прямой), угол HKB=90.
Комментарий: можно также выразить угол DKE через углы DCA и CAE, и применить характеристическое свойство вписанного четырёхугольника (сумма противолежащих углов равна 180).

 

№13.
Опишем около треугольника BEF окружность, отметим её центр O, опустим из O перпендикуляр OK на сторону AC. Пусть точка его пересечения с построенной окружностью—M. Проведём прямую BM и отметим точку её пересечения с AC—K. Она будет искомой. Построение основана на гомотетии.
№14.
Решение: площадь треугольника AIB равна AI*BI*sina/2, где угол AIB равен a. По теореме о сумме углов в треугольнике угол ICB=a-90, но sina=cos(90-a), поэтому CI*sin ICB=CD=a и площадь треугольника AIB, таким образом, численно равна a/2. Так как эта площадь наименьшая из трёх площадей треугольников, на которые его разбивают отрезки AI,BI и CI, площадь ABC> 3a/2, ч.т.д.
 

№15.
Обозначим угол ABC a. Угол BED=90-a, угол DAE=(90-a)/2=угол EDA (теорема о внешнем угле). Угол BAC=90-a/2. Угол DAC=90-a/2- (90-a)/2=45. Ответ: 45.
 
№16.
Заметим, что радиусы  треугольников BHC,AHC,AHB равны радиусу описанной окружности треугольника ABC (так как sinBHC=sin(180-A)=sinA=>BC/sinBHC=BC/sinA=R, аналогично для других сторон). Поэтому шестиугольник AO3BO1CO2- равносторонне-полуправильный, т.е. его диагонали пересекаются в одной точке (обозначим её S). Далее: обозначим углы A, B, C как a,b,c,пусть O1O3 пересекает AB в точке D. Тогда угол BDO1=90-c+90-b=a=> O1O3||AC. Совершенно так же можно доказать, что СO1||AO3. Поэтому отрезки CO3 и AO1  по свойству параллелограмма делятся точкой пересечения пополам. То же можно сказать и о других двух парах отрезков  {BO2 и AO1), (BO2 и CO3).  Если теперь отметить середины треугольника E, F, G на сторонах BC,AC и AB, SE, SF  и SG будут средними линиями  в треугольниках O2BC, O3CB, O1AB (можно выбрать и другие треугольники), следовательно, их длина составит R/2. Тогда очевидно, что S- центр окружности Эйлера.
Есть другой вариант решения, опирающийся на факт, использующийся в доказательстве теоремы о прямой Эйлера и на свойство центра окружности Эйлера.
 



№17.
Решение: так как DF||AE, углы DFB  и EAB равны; поскольку FD и FK- касательные из одной точки,AI-биссектриса угла EAK; тогда, поскольку по теореме о вписанном угле углы EAB и ECB равны, углы DFI и EIH равны, поэтому прямые FI и CG перпендикулярны.  Докажем, что K,J,H,B лежат на одной окружности. Углы BHC и BFC равны, так же, как и углы BDC и BFC (теорема о вписанном угле), значит, углы  BFC и BHC равны, по свойствам симметрии равны углы BEI и IKJ, поэтому угол IKB=90-угол IKJ,  а угол JHB=90+угол BHC, поэтому K,J,H,B  лежат на одной окружности.  Так как MNHL- очевидно, вписанный, то углы ILM=IHM=MBI, значит M принадлежит окружностям, описанным около FDIK  и KJHB. Нетрудно доказать, что F,D,H,B лежат на одной окружности. Тогда MK,FD и BH- радикальные оси трёх попарно пересекающихся окружностей, значит, они пересекаются в одной точке, являющейся их радикальным центром.
 

№18.
NH||OI, биссектриса  AA1 перпендикулярна  OI (так как стороны треугольника составляют арифметическую прогрессию), следовательно, согласно задаче 2012-пв-16, NH-биссектриса угла B1HC.

№19.
Решение: обозначим угол ANC как b, а  угол ANB как a ; тогда угол O2NO1=(b-a)/2 (исходя из свойств касательных из одной точки;  угол O2CA равен  b/2, угол BCA равен a/2 по теореме о вписанном угле;  тогда  угол O2CB равен (b-a)/2, т.е. L, O2, C, N  лежат на одной окружности и O2L перпендикулярен  NO1, следовательно, по свойству диаметра, перпендикулярного хорде, PL=LK.
 

№20.
Решение: нетрудно доказать, что FG||AC;  тогда углы BFG и BIH равны, поэтому B, H, G, I лежат на одной окружности, т.е. углы BHI и BGI  равны; в силу симметрии углы BHI и IHD равны; с другой стороны, углы IBD и BDI и FBD и DBG равны, поэтому ясно, что BG||ID, т.е. углы BGI и GID равны; из этого следует равенство искомых углов.
 
№21.
 Решение: пусть пересечение прямой EG со стороной BC-H; B,H,L,E лежат на одной окружности, так как углы GEC и CBD равны как равные 45 градусам; углы BHE и HED равны как накрест лежащие; по теореме о вписанном угле углы BLC и BFC равны, т.е. B,C,L,F лежат на одной окружности (AFGE-вписанный; так как у него два прямых противоположных угла, значит GEA равен BFC); тогда FL перпендикулярен LC; так как K-ортоцентр треугольника FCE, F,K,L лежат на одной прямой.

№22.
Проведём биссектрису угла AFB; пусть она пересечёт CD в точке J,  а прямые IA и ID- в точках K и L; так как прямая FJ перпендикулярна EI (1), то по теореме о сумме углов треугольника можно доказать, что углы AID и LJD равны, тогда равны углы IKJ и IDJ, а так как DI- биссектриса, то четырёхугольник AKLD-вписанный, и, второй раз применив утверждение 1, получаем искомое.
 

№23.
Решение:
Лемма 1. H,B,I,O  лежат на одной окружности.
Лемма 2. IJ-биссектриса угла HIB, HJ- биссектриса угла BHI.
Лемма 3. K-центр описанной окружности треугольника AJB, L-центр описанной окружности треугольника CJB.
Из этих лемм следует утверждение.
Доказательство леммы 1.
Углы JBF и JAH равны по свойствам равнобедренного треугольника так же, как и углы JBI и JCI (треугольники AFB,AOB,BGC,BOC-равнобедренные). Очевидно также, что равны углы CAO и ACO (как углы при основании равнобедренного треугольника AOC). Следовательно, углы HBO и IBO равны. Обозначим их a. Тогда угол HBI равен 2a, угол AOC=180-2a, отсюда следует утверждение.
Доказательство леммы 2.
Треугольники AOB и COB-равнобедренные, поэтому в силу симметрии ясно,  что OH=OJ=OI Тогда угол JIH=угол HOJ/2=угол HIB/2(теорема о вписанном угле, см. Лемму 1))
Отсюда следует утверждение (во втором случае аналогично)
Доказательство леммы 3.
Так как угол KIB равен  углу KOB, K лежит на окружности, описанной около HBIO. Тогда углы JKF и JBF равны, следовательно F,K,B,J лежат на одной окружности. Тогда, по теореме о вписанном угле, углы KJA и KBF равны, очевидно, что равны и углы KAF и KBF, следовательно, K-центр описанной окружности треугольника AJB (во втором случае аналогично).
На основе этих трёх лемм:
Так как KJ=KB и LJ=LB, отрезок KL перпендикулярен BJ и делит его пополам. Отрезок DE по теореме Фалеса также делит его пополам (ведь DE-средняя линия). Отсюда следует утверждение.

 

№24.
Углы ECF и EDG  равны, так как принадлежат равным прямоугольным треугольникам EFC и EGD (очевидно, что у них  EF=EH, а EC=ED по свойствам прямоугольника.) Тогда угол EIF равен углу EdC по теореме о вписанном угле, значит, угол IEF равен 90-угол EIF=угол EAJ. Поэтому радиус KE перпендикулярен EF, следовательно, CF параллелен KE, следовательно, углы ECF и AEK равны, но углы EAH и  ECF также равны по причине, аналогичной упомянутой в самом начале.
Поэтому касательная AH проходит через центр описанной окружности треугольника AEJ. Тогда угол LAJ равен углу LEJ и поэтому по тереме о вписанном угле LJ перпендикулярен AD.




 


№25 . Указание: используйте следующие утверждения: в тупоугольном треугольнике медиана, проведённая к большей стороне, меньше её половины; сумма длин звеньев ломаной больше длины её одного звена.
 

№26.
Пусть биссектриса пересекает окружность в точке L. Тогда углы EAF и LAH равны как половины угла BAC. Следовательно, углы EAL и FAH равны. Следовательно, EL=FH. Следовательно, LF||AH, следовательно, FL перпендикулярен AF, значит, AL- диаметр окружности.
 
№27. Так как отрезок  OE перпендикулярен BC и DI параллелен AC как средняя линия, прямая DI перпендикулярна OE. Согласно задаче 2011-46 и 2012-52,отрезок FH  перпендикулярен OB; поэтому углы DOG, BFH и BHF равны, значит F,H,G,O лежат на одной окружности.


№28. Решение: углы JDF и HCF равны как соответственные, по аналогичной причине равны  углы JDF и DFC; углы JDF и HDJ равны в силу симметрии, углы ABB1 и ACC1 (HCF) равны как вписанные (так как BB1 и CC1 – высоты). Тогда равны углы ABB1 и DFC, т.е.четырёхугольник FBH’A- вписанный; поэтому углы ABH’ и H’AB1 равны, углы FBC (H’AB1) и JFB равны как соответственные, поэтому четырёхугольник JBFH’-вписанный;  тогда, рассуждая сходным образом, можно доказать, что углы KJE и H’BE равны, а поскольку BB1 перпендикулярен l (параллельной AC), то  JK перпендикулярен BE.
 

№29.
См. рисунок.

.

№30.
Обозначим пересечение O1O2 и BE как F. По теореме о вписанном угле, так как BE-высота и O1O2 перпендикулярен BD (свойства равнобедренного треугольника), sin ADB=sin BFO1 (или, что аналогично, sin EFO1); используя результаты задачи 2012-33, получаем: AC*BE=2O1O2*sin BFO1*BE; AC*BE/2= (1/2*O1O2*sinBFO1*BE)*2, отсюда следует утверждение.
 
№31.
Углы EBD и EDC равны как вписанные, по аналогичной причине равны углы EDC и  EGC (угол ABF- прямой как опирающийся на диаметр, OD перпендикулярен CD как радиус), тогда углы EBD и EGC также равны, поэтому прямые CG и AD перпендикулярны (так как угол ADB- прямой, как опирающийся на диаметр). Обозначим их точку пересечения J. Тогда из того, что F - ортоцентр треугольника ACG следует, что прямая CF перпендикулярна AG. Углы HCG и HDG равны как вписанные (так как у четырёхугольника HCDG- два противолежащих прямых угла), углы JAG и HCG равны по аналогичной причине (AJ и CH-высоты), поэтому углы DAH(JAG) и HDG равны, но так как треугольник AOD- равнобедренный , углы ADH и HAB равны, поэтому AB- касательная к окружности, описанной около треугольника ADH, но, так как IA=ID, I- центр описанной окружности треугольника AHD, т.е. IA=IH=ID, ч.т.д
 
№32.
 Решение: AB || CD, углы ABF и BCF равны как вписанные, следовательно BC-касательная к окружности (A,F,D лежат на одной прямой, потому что угол AFC равен 60 градусам, а угол CBD равен 120 градусам по теореме о вписанном четырёхугольнике и развёрнутом угле); так как O1O2 перпендикулярен CF, углы KO2C и KDC равны, поэтому KDO2C- вписанный;  так как угол CO2D равен 120 градусам, угол FKC равен 60 градусам; но угол AFC равен 60 градусам, следовательно,  KF=FC; поскольку же  AF=BF+FC, AK=BF, ч.т.д.
№33.
Решение: проведём прямую MH и обозначим вторую точку пересечения её с данной окружностью K; докажем, что середина  HK- F- лежит на окружности, описанной около треугольника CMC1. Продолжим CC1 до пересечения с окружностью в точке D. Тогда FC1||KD как средняя линия, по свойству параллельности углы KDH и FC1H равны, по теореме о вписанном угле равны углы KDC и KMC, тогда ясно, что F,С,C1,M лежат на одной окружности. В силу симметричности получаем утверждение задачи.
 
№34. Отметим центр описанной окружности треугольника O. Тогда треуголольник AOC равносторонний, поэтому радиус описанной окружности треугольника равен стороне AC. Известно, что радиус окружности Эйлера треугольника равен половине радиуса его описанной окружности.
Обозначим середину AC как M. Тогда по свойству медианы из вершины прямого угла, так EM=AM=MC, угол AEC—прямой.
№35.
По теореме Нагеля, прямая CO перпендикулярна стороне ортотреугольника A1B1C1 –A1B1 (СС1-высота).Тогда, поскольку прямая CO содержит центр окружности, она содержит и её диаметр, поэтому, по свойству диаметра, перпендикулярного хорде, CO делит DE пополам. Обозначим точку пересечения прямой CO и DE - F.Так как угол CA1B1 равен углу BAC (так как четырёхугольник ABA1B1-вписанный ), то CF=(a-ho)*sinb. Тогда, по теореме о пропорциональности хорд, DE^2/4=CF*(2R-CF). По усиленной теореме синусов, a/sinb=2R. Тогда  отсюда формула.

 
Комментарий: эта же формула верна для любой хорды, параллельной DE.
№36. Ответ: R=scrt(((a+b)^2-a)/2) Указание: примените формулу №57.
№37.
Угол OAC равен 60 градусов, так как треугольник AOC—равносторонний; угол EAC равен углу 45 градусов, так как треугольник AEC—равнобедренный прямоугольный. Тогда угол OAE равен разности углов OAC и EAC и составляет 15 градусов.
№38.
Заметим, что для утверждения задачи достаточно, чтобы AF/FC=AB^2/BC^2. Но AF/CF=AE/CE=AD^2/CD^2=AB^2/BC^2, что и требовалось.

 
№39.
Обозначим AD=c. Очевидно, что CD=c. По теореме Птолемея и теореме косинусов, AB*c+CB*c=BD*c*scrt(2(1+cosa))=>BD=a/scrt(2(1+cosa)).
 

№40.
Первое решение: поскольку OE=EH и по теореме Нагеля прямая AO перпендикулярна отрезку B1C1, LH и AO параллельны и DE- средняя линия. Но H- центр вписанной окружности A1B1C1, поэтому отрезок DE равен (LH+AO)/2. Рассматривая аналогичные DE отрезки, получаем утверждение задачи.
Второе решение: известно, что окружность Эйлера пересекает отрезок BH в такой точке J, что HJ=BJ. Тогда по свойству медианы из вершины прямого угла C1F=FA1, и EC1=EA1. Тогда прямая EJ перпендикулярна A1C1 и параллельна отрезку HK. Тогда JF- средняя линия треугольника HBK, т.е. JF=HK/2. Так как H- центр вписанной окружности треугольника A1B1C1, получаем утверждение.
 
№41. Можно представить контрпример: два равных прямоугольных треугольника, имеющие общую вершину и центрально-симметричные относительно неё, и один из них вращается вокруг этой вершины. Если применить теорему косинусов к двум парам треугольников в получившейся конструкции, непрямоугольных, и заметить что сумма углов, образованных указанными отрезками, для данных треугольников, равна p, то получим доказательство.
Второе решение основывается на перпендикулярности диагоналей четырёхугольника. Вот схема этого решения: из равенства треугольников AFB и CFD следует равенство углов AFB и CFD, поэтому углы AFC и BFD также равны, отсюда из равнобедренности треугольников AFC и BFD следует, что все углы при их основаниях равны. Далее, так как угол CED- прямой, по теореме о вписанном угле, применённой к четырёхугольникам AFEB и DFEC, получаем утверждение задачи.
 



№42.
Первое решение. Через радикальный центр. Заметим, что, так как прямые AB, CD, GE пересекаются в одной точке (K), то, очевидно, G, E, C, D лежат на одной окружности. Обозначим угол EGD a. Тогда угол ABE равен a, а угол ACD равен 180-a и равен углу ABD. Тогда угол ECF равен 180-2a, следовательно, угол ECD равен a, тогда H, F, B, A лежат на одной окружности, отсюда следует утверждение.
 

Второе решение. Через прямые Симпсона (которые не Симпсон открыл). Так как GE- прямая Симпсона, и, по условию, AB, GE и DC пересекаются в одной точке (K), то отрезки AK и DK перпендикулярны. Но тогда, поскольку и HF-прямая Симпсона, то из перпендикулярности указанных отрезков следует, что K принадлежит и этой прямой.
№43.
 
Углы EDB и BEA равны по теореме о вписанном угле, углы EDB и E’D’B равны, так как DE-средняя линия треугольника D’BE’. Тогда по теореме о вписанном угле G, A, D’, B лежат на одной окружности;  проведём через B прямую, перпендикулярную AB. Пусть она пересечёт луч AF в точке I, луч AG- в точке K. Тогда очевидно, что угол ADI равен p/2 по свойствам симметрии. Поэтому IG перпендикулярен AG. Аналогично KF перпендикулярен AF. Тогда H- ортоцентр треугольника AIK и поэтому A,H,B лежат на одной прямой (конкуррентность высот).
№44.
Проведём прямую CI и отметим её точку пересечения со стороной AD F. Так как CI перпендикулярен DI, то  по теореме о вписанном угле несложно доказать, что CE перпендикулярен EF, причём угол ECF равен p/4. Проведём теперь высоту CG. Очевидно, что BC=AG. Также очевидно, что угол EGA равен p/4 (по теореме о вписанном угле, применённой к четырёхугольнику ECGF). Отсюда следует утверждение.
 
Комментарий: если обозначить точку касания окружности и AD как H, то FH=HG. AF=BE (равные прямоугольные треугольники CBE и EAF).
№45.
Решение: обозначим угол ABO a; тогда угол СOD равен углу AOB равен 180-2a, а угол BOD равен 180-a, дальше теорема о смежном угле и признак равнобедренности треугольника

 
№46.
Заметим, что O-центр вписанной окружности треугольника FCG. Действительно, поскольку хорды AC, BC и DE равны и принадлежат одной окружности, точка O равноудалена от них.  Поэтому угол FOG равен 3p/4.

 
№47.
Решение: заметим, что DE параллелен BC (свойство, обратное свойству параллельных хорд); тогда по свойству параллельности, DF перпендикулярен AF.  По теореме о вписанном угле, OE перпендикулярен AB. Тогда OF-прямая Симпсона. Пусть она пересекает  BC в точке H. Тогда EH перпендикулярен BC. По теореме о вписанном угле  угол HOE равен углу HBE, и, как нетрудно посчитать, равен p/12. Пусть точка пересечения OE и BC-K.  Тогда ясно, что четырёхугольник  ACKO- вписанный, тогда угол COK равен p/6, значит угол COH равен углу COG равен p/12, тогда угол AGO равен углу AGC равен 5p/12. По теореме о сумме углов в треугольнике, применённой к треугольнику ACG, и по признаку равнобедренности треугольника, AC=AG.
 




№48.
Решение: обозначим угол BCA 3a. Тогда угол ADC=2a, по теореме о сумме углов треугольника, применённой к равнобедренному треугольнику ADC, угол DAC равен a. По этой же теореме, применённой к прямоугольному треугольнику BAC, угол BAC равен 90-3a. Тогда угол BAD=углу BAE=180-4a. Тогда по той же теореме, применённой к равнобедренному треугольнику BAE, угол AEB равен 2a. Тогда, по признаку параллельности прямых, отрезок BE параллелен отрезку CD.
 


№49.
Ответ: 2a(scrt(scrt5-1)-scrt(scrt10-2scrt2). Для вывода этого выражения нужно применить теорему Птолемея к вписанному четырёхугольнику CIE’A. Доказательство вписанности: угол AEC=угол AED+ угол DEC=3p/4=> угол AE’C= углу AEC=углу AIC, теорема о вписанном угле.
 

Комментарий: можно ещё решить более тривиально, например: найти EI, EE’ и угол E’EI. Для нахождения первого нужно поступить следующим образом: обозначить точку пересечения EF (напр., как H), точку пересечения IE (напр., как L), точку касания вписанной окружности треугольника ABC (например, как K; использовать подобие треугольников LIK и LEH и теорему Пифагора; использовать факт, что B, E, I лежат на одной прямой и применить формулу косинуса разности углов и теорему косинусов для треугольника IEE’.
№50. Указание: доказать формулу r=ah/2(a+h) и применить a>h к знаменателю, затем сократить a.
№51.
Пусть угол ACC_1 равен a. По теореме о внешнем угле, угол EC_1C равен a/2. Ясно, что угол BC_1E равен  90-a/2, а угол ABB_1 равен a. Тогда, по теореме о сумме углов в треугольнике, применённой к треугольнику C_1BF, углы BC_1F и BFC_1 равны, значит, по признаку равнобедренности треугольника, BC_1=BF.
№52.
Из подобия треугольников AED и ADB и CFD и CDB следуют равенства AD^2=AE*AB и CD^2=CF*BC. Но это значит, что AC—касательная к окружности, описанной около четырёхугольника BEDF. Ясно, что G—центр этой окружности. Тогда GD и AC перпендикулярны, но D—середина AC, значит, в треугольнике AGC высота GD является также медианой, значит, он равнобедренный. Поэтому AG=CG.
№53.
Согласно задаче 2010 года, BJ параллелен AC. Ясно также, что четырёхугольник AFJE—вписанный ( у него два противоположных прямых угла). Обозначим угол BAC как a. Тогда угол AJE равн 90-a/2; по теореме о вписанном угле, угол AFE также равен 90-a/2, а угол JBF равен a как сответственный углу BAC при секущей AB параллельных прямых BJ и AC.  Поэтому по теореме о сумме углов в треугольнике, применённой к треугольнику EBH, угол BEH равен 90-a/2, т.е.BH=BE.
№54.
Угол AED равен углу AFD, так как FAED- параллелограмм; углы AED и AE’D равны по свойствам симметрии, значит, A, F, E’, В лежат на одной окружности. А так как DE’=DE=AF, то AD параллелепараллелен EF.
Второе решение: обозначим пересечение AD и EF как G; тогда E’G=GE и FG=GE, значит, угол FE’E- прямой, тогда отрезок EF параллелен медиане AD.
 
№55.
1) Проведём BD и обозначим точку пересечения отрезков BD и  CE I.
2) Проведём прямые IG и BC и обозначим точку их пересечения J.
3) Проведём прямую JA и окружности как M.
4) Проведём прямые MG и MH и обозначим их точку пересечения и  окружности K и L.
5) Строим KL.
Задача основана на свойстве радикального центра.
 

№56.
Обозначим угол BDA как a. Тогда, по теореме о вписанном угле, углы BAO_1 и BCO_2 равны 90-a и следовательно, равны друг другу. По этой же теореме получаем, что точка пересечения прямых AO_1  и CO_2 лежит на описанной окружности треугольника ABC.
№57.
По свойству медианы из вершины прямого угла, углы FBC и FCB равны. Угол FDE равен углу FCE по теореме о вписанном угле. По этой же теореме, угол FHE равен 180-угол FCE. Угол FBE равен 180-угол FBC, как смежный самому углу FBC. Поэтому углу FHE и FGE равны
№58. Пусть точка пересечения диагоналей четырёхугольника O_1ABO_2—E.  Так как трезки  O_1A и O_2B параллельны, углы O_1AO_2 и BO_2A равны. Очевидно также, что углы O_1EA и O_1AO_2 равны (как углы при основании равнобедренного треугольника). Углы же BEO_2 и O_1EA равны как вертикальные. Тогда по признаку равнобедренности треугольника, треугольник EBO_2—равнобедренный. Поэтому отрезки O_1O-2 и O_1B оба равны сумме радиусов окружностей. Опутим из точки O_1 перпендикуляр O_1K на отрезок O_2B. Тогда O_1ABK—прямоугольник и, по его свойству, O_1A=KB. Но по свойству равнобедренного треугольника, O_K является также медианой треугольника O_1BO_2.
Тогда искомое соотношение радиусов 1 к 2.
№59.
AE/CE=AO/OC=AD/BC (1) по свойству биссектрисы угла треугольника и по свойствам подобия (пара подобных треугольников AOD и BOC).HE/CE=AE/BE=AH/BC (2) по свойствам подобия (пара подобных треугольников HAE и CBE).Из (2) следует, что AE*CE=HE*BE (3).  Перемножаем самые левые части (1) и (3) и получаем: AE*CE*(AE/CE)=(AO/OC)*HE*CE (4), отсюда AE^2=(AO/OC)*HE*CE (5) Применив (1) получим:AE^2 = (AD/BC)*HE*BE (6)Преобразуем равенство (6): (AE/BE)*AE=(AD/BC)*HE (7)Применив (1) и (2) получим: (AH/BC)*AE=(AD/BC)*HE (8) Отсюда AH*AE=AD*HE. Дальнейшее ясно.
№60.
Так как углы DAB и EAC равны и составляют 6о градусов, то углы DAE и BAC равны, следовательно, треугольники ADE и ABC равны и угол ADE равен 90 градусам. Поэтому треугольник ADM—прямоугольный с углом 60 градусов. Значит,  AD=AB=AM/2, т.е. точка M симметрична A относительно B.
№61.
По теореме о вписанном угле, углы BAF и BEF равны. Углы EBC и ABF равны по этой же причине. Углы ABO и BAO равны как углы при основании равнобедренного треугольника. Тогда углы BEF и EBC равны, и из свойства медианы из вершины прямого угла, следует, что прямая EF проходит через середину стороны BC.
№62. Строим BD. Возьмём любую точку на отрезке AD—E и восстановим в ней к отрезку BD перпендикулярный отрезок, находящийся внутри ABCD—EF. (1) Повторим ту же операцию для построенного отрезка, так что концы отрезка GH, перпендикулярного отркезку EF, ледат на стронах четырёхугольника AC и CD соотвественно(2). Строим отрезки DG и AH и обозачим точку пересечения их K (3,4). Проводим через C и K часть луча CK до пересечения  с отрезком AC в точке M (5). Перпендикуляр к отрезку AC в точке M пересечёт CD в искомой точке (6).
В построении использовано замечательное свойство трапеции.
№63.
См. рисунок.

 

№64.
По свойству прямоугольного треугольниа с углом 30 градусов, CD=CF. Угол BCD  равен 120 градусам как смежный углу ECB, равному, очевидно, 60 градусам.  Тогдв треугольники  FCB и DCB равны по двум сторонам и углу между ними. Отсюда следует равенство их сторон BF и BD.

№65.
Угол BAC равен углу DBC, а углы  OCB и BCD равны разности прямого угла и каждого из углов BAC и DBC  соответственно. Поэтому прямая CO содержит точку E.
№66.
Углы ABC и DBC равны по известному свойству равнобедренного треугольника. Углы  CED и CBD равны по теореме о вписанном угле, так же, как и углы ABC и CEF. Следовательно, углы CEF и CED равны, откуда следует утверждение.
№67.
Пусть меньшая окружнсть, расположенная внутри круга, будет называться окр.1, а большая—окр.2. Обозначим центр окр. 2 как O, точки пересечения прямой OI с большой окружностью как D и E, причём D лежит в одной полуплоскости относительно прямой AB, что и O. Обозначим точку касания касательной из точки A с окр.2 как C, а точку касания самих окружностей (окр.1 и окр.2) как F.
Обозначим радиус окр.1 как x, а радиус окр.2—как y. Заметим, что углы AIF и CAF равны (OC||AI, а четырёхугольник ACOF—вписанный). Тогда по теореме о пропорциональности хорд окружности, AB можно выразить через x и y. Также можно выразить через x и y тангенс угла AIF, а значит, и тангенс угла CAF . Зметим, что AC=AB/2 и из четырёхугольника ACOF теперь модно найти OF, т.е. составится уравнение.
Искомое отношение будет 1:2.
№68.

 

Разобьём задачу на две. Сначала докажем, что угол CAF равен половине угла OHG. Во-первых, точки A, O, G, D лежат на одной окружности, следовательно, угол AGD равен 90 градусов, по теореме о вписанном угле. Следовательно, углы OAG и ODG равны. С другой стороны, по свойству симметрии равны и углы OAM и ODM. Таким образом, угол CAF равен половине угла MDG. Также очевидно, что точки H,M,G,D лежат на одной окружности, а значит, угла MDG и MHG равны. Значит, угол CAF равен половине угла OHG.
 

Теперь докажем, что  угол CAI равен половине угла CLI. Из этого будет следовать утверждение задачи, так как точка I принадлежит лучу AF. Так как отрезок OK перпендикулярен AB, точки  O,K,L,A лежат на одной окружности по теореме о вписанном угле. Из этого следует, что угол KJA равен 135 градусам, угол же LDA равен 45 градусам, а значит,  точки D, L, J, A лежат на одной окружности. Ясно, что на ней лежит и точка I; тогда IL=LA=CL, т.е. точка L является центром описанной окружности треугольника CAI.  Значит, так как вписанный в окружность угол равен половине её центрального угла, угол CAI равен половине угла CLI.
 

№69.
Пусть угол JCD равен a. Продолжим отрезки HJ и AJ до пересечения с прямой CD в точках K и N.  Тогда угол AND равен 90-a, а так как угол HJC, как нетрудно доказать, прямой, то угол NJK, по теореме о сумме углов в треугольнике, равен 2a. Значит, угол HJA также равен 2a как вертикальный ему (ясно также, что углы HCA и JCD равны). Тогда в силу симметрии точка J—центр описанной окружности треугольника HCA.
Очевидно также, что точка F является центром описанной окружности треугольника ECA. Тогда углы HJA и EFA равны. Обозначим точку пересечения прямой JH и отрезка EF как P. Понятно, что угол FAB равен половине угла EFA,  и поэтому равен углу AJD. Тогда отрезки FA и AJ перпендикулярны так же, как и отрезки JP и PF.
Угол FCA равен (90-a),а угол CED равен (45-a), поэтому угол FEC равен 45 градусам, и точки E,F,C,D лежат на одной окружности, откуда, по теореме о впианном угле, отрезки AF и CF перпендикулярны. Отсюда JH и CF параллельны, поэтому углы HJF и JFC равны, также равны углы HJB и HCB , так как точки H,B,C,J лежат на одной окружности. Углы AFG и СFG равны по свойству симметрии.
Тогда четырёхугольник IFCG—вписанный. G—центр описанной окружности треугольника ICA. Следовательно, угол ICA равен половине угла IGA, а так как он сам равен 45 градусам, угол IGA равен 90 градусам. Поэтому угол IAB равен углу IJB.
Значит, углы JCD и IGB равны.
Комментарий: имеются ещё варианты решения, но они в общем близкой сложности.

 
№70.
Обозначим угол EBC как a. Угол ECB равен 45-a по теореме о сумме углов в треугольнике По теореме о смежном угле, угол ECD равен 45-a, по теореме о сумме углов в треугольнике, угол CDG равен 45-a. Угол AGD, очевидно, равен 45 градусам. Обозначим пересечение отрезка AG и стороны квадрата CD как K.
Тогда угол AKD равен 90-a. Угол EDC и EBC равны по теореме о вписанном угле. Поэтому отрезки DE и AG перпендикулярны. Диагонали квадрата также перпендикулярны. Тогда AOHD—вписанный. По теореме о вписаннм угле, углы DOH и GAD равны. Тогда очевидно равенство углов DOH и BEC.
К этому равенству добавится и угол HGF, если учесть вписанность четырёхуольника HEFG.

№71.
Опишем около треугольников EAO и CDO окружности и обозначим их окр.1 и окр.2 соотв. Описанную окружность квадрата обозначим окр.3. Тогда точка F—их радикальный центр, следовательно, окр.1 и окр.2 пересекаются на отрезке FO в некоторой точке G’. Проведём отрезки OE и CE и обозначим угол ECA как a. Тогда угол AOE будет равен по теореме о вписанном угле 2a, угол AEO—(90-a), угол EBC—(45-a) по теореме о сумме углов в треугольнике (рассматриваются треугольники BOE и BEC). Далее, по теореме о вписанном угле, угол BG’O равен (90-a), угол  BG’E равен 2a, угол DG’O равен 45 градусам, угол CG’D равен 90 градусам. Из этого получаем, что сумма углов BG’O и DG’O равна (135-a) и равна таким образом углу EBA. Пользуясь же тем, что углы BG’O, DG’O, BG’E, CG’D и EG’C в сумме, собственно, составляют 360 градусов, получаем и то, что углы BG’D и EG’C равны, и, таким образом, точки G’ и G совпадают, что и требовалось доказать



 

№72.
Пусть угол CFD равен b. Тогда угол LGC равен (135-b). Тогда угол LEC больше величины (45+b). Угол же BEC равен 135. Нетрудно также посчитать, что угол EBС равен величине (90-b). Угол BEL меньше величины (90-b), следовательно, угол BEL меньше угла EBC. Так как отрезки BE и ED перпендикулярны, из вышесказанного следует что угол EIL—острый. Отсюда следует, что угол ECL меньше 45 градусов,  и EL<CD. Следовательно, так как стороны AD и BC параллельны,угол LDA больше угла ECB.
По теореме о вписанном угле, угол BLE равен углу BCE. Как уже было выше сказано, угол ECI (ECL) меньше 45 градусов, значит, и угол УВД меньше 45 градусов, а угол ELO (MLO) больше 45 градусов. Опишем около треугольника BEM окружность; пусть её точка пересечения  с отрезком BD—P. Угол MLP равен углу MBP и равен 45 градусам (ведь угол MBP равен углу между стороной и диагональю квадрата). Значит, точка P принадлежит отрезку BO, и , по теореме о внешнем угле, угол BOM меньше угла BPM, а значит, и угла BLM.
Т.е., угол BOM меньше угла ADM, что и требовалось установить.







 

№73.
По свойству радикального центра (см.3. №5)  можно найти, что точки B, F, G, O лежат на одной окружности. Из задачи №5 углы ABE и EGL равны тогда по теореме о вписанном угле, на той же окружности лежит и точка H. Так как угол BCE равен половине угла BOE, а угол BHE равен углу BOE, и отрезки BO и OC перпендикулярны, углы EOC и BOC равны. Тогда равны и углы IOE и ICE. Поэтому углы BOJ и  BAJ равны. Из этого следует, что отрезки BJ и AJ перпендикулярны. Угол BEA равен, очевидно, 45 градусам. Значит, из теоремы о сумме углов в треугольнике (применённой к треугольнику BJE) следует, что  угол EBJ (EBG) также равен 45 градусам. Тогда, так как прямая OI—ось симметрии в равнобедренном треугольнике BOE, отрезки BG, IO и AE пересекаются в одной точке, что и требовалось доказать.

 




№74.

Точки A, G, D, C лежат на одной окружности, так как прямая GD отсекает от треугольника ABC равнобедренную трапецию. Пусть P-точка пересечения прямых l и BE.  Так как углы BCA  и BCP равны и углы BCA и BEA также равны, четырёхугольник CDEP—вписанный. Так как пары углов BCE, DPE и GAE, BCE равны, углы GAE и GPE также равны, т.е. точки A, G, E, P лежат на одной окружности.
Тогда, по свойству радикального центра, прямые  CD, AG и PE пересекаются в одной точке, но это и будет означать, что прямая GD проходит через точку P.
№75.
 Заметим, что прямые AE и DE пересекают любой луч, проведённый из точки B и пересекающий сторону CD. Поскольку угол AED—тупой, то угол BEG—тоже тупой, как больший угла AED. Легко доказать, что угол AFD меньше 45 градусов. Тогда, применив параллельный перенос, получаем, что угол BGC меньше 45 градусов. Ясно тогда, что угол BFC меньше половины угла AED.
№76
Углы DBO и ODE равны по теореме об угле между хордой и касательной. Углы DBO и EBO равны по свойству симметрии. Следовательно, углы ODE и FBO равны. Так как прямая BO перпендикулярна DE по свойству симметрии, по теореме о вписанном угле, прямая DO перпендикулярна отрезку BF.

№77.
А) Так как угол ECA равен 45 градусам, то по теореме о вписанном угле четырёхугольник AECG- вписанный, поэтому его угол ECG- прямой. Значит, угол DCG равен 45 градусам. Также угол DCP равен 90 градусам. Поэтому треугольник ECG- прямоугольный равнобедренный, и угол DGK равен 45 градусам. Опустим перпендикуляр PR на сторону BC. Тогда PR=RK. Отметим середину отрезка CF- T, также отметим центр описанной окружности треугольника CEF- O. Так как угол COF- прямой по теореме о вписанном угле и треугольник CEF- равнобедренный, причём ET- его линия симметрии, то OT=CF/2. По теореме Эйлера, EP=2OT, тогда BR=2BK и BE=BK, ч.т.д.
Б) Пусть точка пересечения MK и AC в точке M_1; MM_1=M_1K как половины гипотекузы KM и MM_1=ON как высоты равнобедренного треугольника OCM. Значит, KM=CF.
 

№78.
По теореме Менелая, применённой к треугольнику AEO, OJ/JE*EF/AF*AM/MO=1
OJ/JE=OM/AM. Согласно Ефремову (книга «Новая геометрия треугольника», вышедшая в Одессе), JO/JE=IO/IE. Поэтому OM/AM=OI/IE. Т.е., IM||AE (теорема Фалеса). Так как отрезок GJ—средняя линия треугольника AED, GJ||AE. Поэтому, по известной теореме о трёх параллельных прямых, IM||GJ. Применим замечательное свойство трапеции сразу к двум трапециям—AEIM и JMIG. Тогда и отрезок OF, и отрезок KL пройдут через середину отрезка IM. Тогда все эти три отрезка пересекаются в одной точке.

Заключение. На этом планиметрический дневник завершается.


© Copyright: Николай Москвитин, 2015